You are on page 1of 73

At which of the following ages does fetal movement first occur? A. 1 month B. 2 months C. 4 months D. 6 months E.

7 months

The correct answer is B. Neuromuscular development is sufficient to allow fetal movement in the eighth week of life. Other features of Week 8 include the first appearance of a thin skin, a head as large as the rest of the body, forward-looking eyes, appearance of digits on the hands and feet, appearance of testes and ovaries (but not distinguishable external genitalia), and a crownrump length of approximately 30 mm. By the end of the eighth week, nearly all adult structures have at least begun to develop, and the fetus "looks like a baby."

Most of the oocytes in the ovary of a prepubescent girl are in which meiotic stage? A. Anaphase of the second meiotic division B. Metaphase of the first meiotic division C. Metaphase of the second meiotic division D. Prophase of the first meiotic division E. Telophase of the first meiotic division

The correct answer is D. The first meiotic division is the "reduction" meiotic division, in which the diploid complement of DNA is reduced to a haploid complement. The bulk of oocytes in premenopausal women, girls, and babies are arrested at prophase of the first meiotic division. Postmenopausal women have very few viable oocytes. It is important to note that ovulation occurs before the oocyte is completely mature. The secondary oocyte leaving the follicle is in metaphase of the second meiotic division (choice C). The cell's metabolic operations have been discontinued and the oocyte drifts in a state of "suspended animation," awaiting the necessary stimulus for further development. If fertilization does not occur, the oocyte disintegrates without

completing meiosis.

An abrasion results in the total loss of epidermis over a large area of an arm, but one month later, the abrasion has healed, with regrowth of the epidermis. Which of the following mechanisms accounts for the restoration of the epidermis over the abraded area? A. Growth of epidermis from hair follicles and sweat glands in the dermis B. Migration of endothelial cells from newly grown capillaries C. Transformation of dermal fibroblasts into epidermal cells D. Transformation of macrophages into epidermal cells E. Transformation of melanocytes into epidermal cells

The correct answer is A. The dermis contains skin appendages (e.g., hair follicles), which contain epithelial stem cells. In the process of healing a large area where the epidermis has been lost but the dermis is intact, re-epithelialization occurs by growth of epidermal cells from the underlying skin appendages, as well as from the intact epidermis along the wound edges. Physiologically, the dermis lies beneath the epidermis. It has two major components, a superficial papillary layer and a deeper reticular layer. The papillary layer contains the capillaries and the sensory neurons, which supply the surface of the skin. The reticular layer consists of an interwoven meshwork of dense irregular connective tissue. None of the other cell types are known to directly contribute to the regeneration of epidermis over abraded skin.

Hirschsprung's disease indicates a developmental abnormality in which of the following embryonic tissues? A. Ectoderm B. Endoderm C. Neural crest D. Neural ectoderm

E. Splanchnic mesoderm

The correct answer is C. The baby has Hirschsprung's disease, which is due to an absence of ganglion cells in the wall of the colon. Neural crest cells contribute to the formation of many adult structures. Among these are all of the postganglionic neurons of the autonomic nervous system and the sensory neurons of the peripheral nervous system. Ectoderm (choice A) forms the epidermis of the skin and the parenchymal cells of glands associated with the skin such as the sweat glands, sebaceous glands, and mammary glands. Endoderm (choice B) forms the epithelial lining of the gut tube and the parenchymal cells of glands associated with the gut tube, such as the liver and pancreas. Neural ectoderm (choice D) forms the central nervous system, the somatic motor neurons of the peripheral nervous system, and the preganglionic neurons of the autonomic nervous system. Splanchnic mesoderm (choice E) forms the visceral peritoneum, visceral pleura, visceral pericardium, and the stroma and muscle of the wall of the gut, among other structures.

Injury at the lower border of a rib will most likely damage which structure? A. intercostal artery B. intercostal nerve C. intercostal vein D. internal intercostal muscle

The correct answer is B. The three structures in the intercostal space are, from superior to inferior, the intercostal vein, artery, and nerve. The proper site for insertion of an intercostal drain is superior to a rib, not directly at the level of the superior border but slightly higher to avoid the collateral branches of the nerve, artery, and vein. The nerve is the most inferior structure and thus most likely to be damaged by the drain (producing an anesthetic dermatome). In addition, the nerve is the least protected by the costal groove. The intercostal artery (choice A) is the middle structure and thus is not the most likely to be damaged.

The intercostal vein (choice C) is the most superior structure and is thus the least likely to be damaged. In addition, the vein is the structure best protected by the costal groove (the further superior the structure, the more protected by the costal groove). The internal intercostal fibers (choice D) will tend to be separated by the drain. In addition, the internal intercostal muscle is membranous (internal intercostal membrane) posteriorly, from the neck of the rib to its angle; therefore, many drains, when inserted, will not even pass through a layer of internal intercostal muscle.

Following the healing of a tibial fracture, a patient forms a small, new piece of viable bone near the joint (heterotopic ossification), which is interfering with joint mobility. Which of the following is the source of the osteoblasts that formed this bone? A. Cancellous bone near the fracture B. Circulating stem cells C. Compact bone near the fracture D. Marrow E. Periosteum

The correct answer is E. The source of bone-forming cells following a fracture is the damaged periosteum. Occasionally, some of these cells become "lost" and can form small pieces of inappropriately located bone that impair joint movement. Except in joint cavities, where they are covered by a layer of hyaline cartilage, bone surfaces are covered by the periosteum. The periosteum is composed of an outer fibrous and inner cellular layer. The periosteum assists in the attachment of a bone to surrounding tissues and associated tendons and ligaments. Surprisingly, neither the cancellous (choice A) nor the compact (choice C) bone near the fracture is the source of the bone-forming cells. Circulating stem cells (choice B) can repopulate damaged bone marrow (choice D), but neither the bone marrow nor the circulating stem cells is the source of the bone-forming cells.

A patient has a severe headache, and is unable to move his right leg. There is no higher cortical function loss, but deep deep tendon reflexes and Babinski's sign are absent. The medial aspect of the cerebral hemispheres is most affected. Which artery is most likely involved?

A. left anterior cerebral artery B. left middle cerebral artery C. left posterior cerebral artery D. right anterior cerebral artery E. right posterior cerebral artery

The correct answer is A. The medial aspect of the cerebral hemispheres is supplied by the anterior cerebral arteries. The small portion of primary motor cortex located here sends fibers to spinal-cord segments innervating the lower limbs. Because the corticospinal tracts cross over to the opposite side in the medulla oblongata, the left cerebral hemisphere controls the right side of the body. Hence, paralysis of the right leg results from loss of blood flow to the portion of the left hemisphere supplied by the left anterior cerebral artery. The middle cerebral artery (choice B) supplies the lateral convexity of the brain. Primary motor cortices on the lateral aspects of the hemispheres send fibers to the brainstem (innervating the face) and to spinal-cord segments innervating the upper limbs, the trunk, and the proximal part of the lower extremities. As with the anterior cerebral artery territories, the right hemisphere innervates the left body, and the left hemisphere innervates the right body. The posterior cerebral artery (choices C and E) supplies the occipital cortex; interruption of blood flow through this artery would probably produce visual deficits rather than paralysis. The right anterior cerebral artery (choice D) supplies blood to the medial aspect of the right cerebral hemisphere, which controls the left leg. This patient has paralysis of the right leg.

An infant is born with an abnormally developed falciform ligament. The hepatogastric and hepatoduodenal ligaments are also malformed. These developmental anomalies are most likely due to abnormal development of the A. dorsal mesoduodenum B. dorsal mesogastrium C. pericardioperitoneal canal D. pleuropericardial membranes E. ventral mesentery

The correct answer is E. The ventral mesentery forms the falciform ligament, ligamentum teres, and lesser omentum, which can be divided into the hepatogastric and hepatoduodenal ligament. The dorsal mesoduodenum (choice A) is the mesentery of the developing duodenum, which later disappears so that the duodenum and pancreas lie retroperitoneally. Both omental bursa and the greater omentum are derived from the dorsal mesogastrium (choice B), which is the mesentery of the stomach region. The pericardioperitoneal canal (choice C) embryologically connects the thoracic and peritoneal canals. The pleuropericardial membranes (choice D) become the pericardium and contribute to the diaphragm.

Which of the following structures is lined with epithelium derived from mesoderm of the ureteric bud? A. Bowman's capsule B. Distal convoluted tubule C. Loop of Henle D. Proximal convoluted tubule E. Ureter

The correct answer is E. The transitional epithelium that lines the ureter, the renal pelvis, and the major and minor calyces is derived from mesoderm of the ureteric bud, as is the cuboidal epithelium of the collecting tubules. The simple squamous epithelium lining Bowman's capsule (choice A) is derived from mesoderm of the metanephric vesicle. The simple cuboidal epithelium lining the distal convoluted tubule (choice B) is derived from

mesoderm of the metanephric vesicle. The simple squamous epithelium lining the loop of Henle (choice C) is derived from mesoderm of the metanephric vesicle. The proximal convoluted tubule (choice D) is lined with simple columnar epithelium derived from mesoderm of the metanephric vesicle.

A patient's left hypoglossal nerve (CN XII) is injured during a carotid endarterectomy. Which of the following would most likely result from this injury? A. Decreased gag reflex on the left B. Decreased salivation from the left submandibular and sublingual salivary glands C. Deviation of the tongue to the left on protrusion D. Inability to elevate the pharynx on the left during swallowing E. Inability to perceive sweet and salt taste sensation on the anterior part of the left side of the tongue

The correct answer is C. The hypoglossal nerve is a pure motor nerve (general somatic efferent) to the intrinsic and most extrinsic muscles of the tongue. If the nerve is damaged, denervation atrophy of the affected side will permit the intact musculature of the opposite side to operate unopposed, thereby protruding the tongue to the side of the injury. The gag reflex (choice A) is mediated by the glossopharyngeal nerve (CN IX; afferent limb) and the vagus nerve (CN X; efferent limb). Choice B is incorrect because the preganglionic parasympathetic fibers that regulate these two salivary glands are carried by the chorda tympani (which joins with the lingual nerve) to the submandibular ganglion. Postganglionic fibers are then distributed to these glands. The muscles responsible for elevation of the pharynx (choice D) are innervated primarily by the vagus nerve (CN X). Choice E is incorrect because the taste fibers for the anterior two thirds of the tongue are carried via the chorda tympani to the facial nerve (CN VII) and hence to the brainstem.

A patient with paresthesia of the medial side of the hand and forearm undergoes surgery to correct it. If the anterior scalene muscle is cut during this surgery, what nearby nerve must be avoided? A. Phrenic nerve B. Recurrent laryngeal nerve

C. Superior laryngeal nerve D. Suprascapular nerve E. Vagus nerve

The correct answer is A. The patient is suffering from scalene triangle syndrome (thoracic outlet syndrome). The lower trunk of the brachial plexus and the subclavian artery are being compressed between the anterior scalene muscle and the middle scalene muscle. Incision of the anterior scalene muscle will relieve this compression. The phrenic nerve lies on the anterior surface of the anterior scalene muscle deep to the prevertebral fascia. If this nerve is cut, the diaphragm on that side of the body will be paralyzed. Anatomically, the phrenic nerve is one of the nerves of the cervical plexus. This nerve distributes to the diaphragm other nerves of the cervical plexusm including the ansa cervicalis, lesser occiptital, transverse cervical, supraclavical, and greater auricular nerves, as well as the cervical nerves. The recurrent laryngeal nerve (choice B) is a branch of the vagus nerve. On the left side it recurs around the aortic arch, and on the right side it recurs around the right subclavian artery. It ascends to the larynx in the tracheoesophageal groove. The nerve innervates several laryngeal muscles and the laryngeal mucosa inferior to the vocal folds. It is not in contact with the anterior scalene muscle. The superior laryngeal nerve (choice C) is a branch of the vagus nerve that arises just after the vagus nerve passes through the jugular foramen. It innervates the cricothyroid muscle of the larynx and the laryngeal mucosa above the vocal folds. The superior laryngeal nerve is not in contact with the anterior scalene muscle. The suprascapular nerve (choice D) is a branch of the upper trunk of the brachial plexus. It arises from the upper trunk after the upper trunk has passed between the anterior and middle scalene muscles. The suprascapular nerve innervates the supraspinatus and infraspinatus muscles. The vagus nerve (choice E) lies within the carotid sheath within the neck. The carotid sheath is anterior to the prevertebral fascia. The vagus nerve is not in contact with the anterior scalene muscle.

Compression of a cranial nerve by a large aneurysm of the right superior cerebellar artery immediately distal to its origin from the basilar artery would cause which of the following clinical findings? A. Loss of abduction of the right eye B. Loss of adduction of the right eye C. Loss of depression of the right eye from the adducted position D. Loss of sensation on the right side of the face E. Loss of visual field of the right eye

The correct answer is B. The oculomotor nerve emerges from the interpeduncular fossa of the midbrain and then passes between the superior cerebellar artery and the posterior cerebral artery immediately lateral to the basilar artery. Aneurysm of any of these three arteries may compress the nerve. The oculomotor nerve innervates a number of extraocular muscles in the orbit, including the medial rectus muscle, which is responsible for adduction of the eye. Abduction of the eye (choice A) is accomplished by the lateral rectus muscle. This muscle is innervated by the abducens nerve, which arises more caudally from the brainstem and is not in contact with this artery. Depression of the eye from the adducted position (choice C) is accomplished by the superior oblique muscle. This muscle is innervated by the trochlear nerve, which emerges from the dorsal surface of the midbrain and passes lateral to the cerebral peduncle before entering the cavernous sinus. A superior cerebellar artery aneurysm would not compress this nerve. Sensation on the face (choice D) is mediated by the trigeminal nerve. This nerve arises from the anterolateral surface of the pons and is not in contact with this artery. Loss of visual field (choice E) results from lesions of the optic nerve or other elements of the visual pathway. These are not in contact with this artery.

The two most important hormones controlling Calcium metabolism are secreted from the: A. Parathyroid and thyroid glands B. Thyroid and thymus glands C. Adenohypophysis and parathyroid glands

D. Adrenal cortex and thyroid glands E. Parathyroid and adrenal glands

The correct answer is A.PTH, or parathyroid hormone, is considered to be the most important hormone controlling calcium metabolism. It increases serum calcium by increasing bone mineral resorption by osteoclasts, increasing digestive absorption of calcium, and decreasing calcium excretion by the kidney. PTH is secreted by principal cells of the parathyroid. The antagonist hormone, calcitonin, is secreted by parafollicular cells of the thyroid (follicular cells secrete thyroxin). It reduces serum calcium by promoting calcium depositon in bones, decreasing digestive absorption of calcium, and promoting calcium excretion. Note that the term adenohypophysis in answer choice C refers to the anterior pituitary. This term and neurohypophysis (posterior pituitary) are often used on NBDE.

During a cranial nerve test, the patient cannot elevate her right eye from the abducted position. Which of the following muscles is paralyzed? A. Right inferior oblique B. Right inferior rectus C. Right lateral rectus D. Right superior oblique E. Right superior rectus

The correct answer is E. The superior rectus muscle can elevate and adduct the eye from the neutral position. From the abducted position, it is the only muscle that can elevate the eye. The inferior oblique muscle (choice A) can elevate and abduct the eye from the neutral position. From the adducted position, it is the only muscle that can elevate the eye. The inferior rectus muscle (choice B) can depress and adduct the eye from the neutral position. From the abducted position, it is the only muscle that can depress the eye. The lateral rectus muscle (choice C) can abduct the eye. The superior oblique muscle (choice D) can depress and abduct the eye from the neutral

position. From the adducted position, it is the only muscle that can depress the eye.

Several arterial branches penetrate into the distal end of the lesser curvature of the stomach. Which of the following arteries usually supplies these branches? A. Left gastric B. Left gastroepiploic C. Right gastric D. Right gastroepiploic E. Short gastric

The correct answer is C. The arterial supply of the stomach is complex; it therefore has a good potential to appear on the NBDE. The right gastric artery supplies the distal lesser curvature. The left gastric artery (choice A) supplies the proximal lesser curvature. The left gastroepiploic artery (choice B) supplies the proximal greater curvature below the splenic artery. The right gastroepiploic artery (choice D) supplies the distal greater curvature. The short gastric artery (choice E) supplies the proximal greater curvature above the splenic artery.

Contraction of which of the following muscles contributes most to the backward movement of the lower jaw during the process of mastication? A. Digastric B. Lateral pterygoid C. Medial pterygoid D. Mylohyoid E. Temporalis

The correct answer is E. Mastication is a complex process involving alternating elevation, depression, forward movement, and backward movement of the lower jaw. The backwardmovement step is accomplished by the posterior fibers of the temporalis muscle. The temporalis originates along the temporal lines of the temporal bone and inserts into the coronoid process of the mandible. It is innervated by the manibular branch of the trigeminal nerve. The digastric (choice A) helps to depress (open) the lower jaw during chewing. The lateral pterygoid (choice B) helps to move the lower jaw forward, laterally, or downward during chewing. The medial pterygoid (choice C) helps to elevate (close) the lower jaw during chewing. The mylohyoid (choice D) helps to depress (open) the lower jaw during chewing.

Most fascia of the body that attach to bones attach by which of the following mechanisms? A. Blending with the covering periosteum B. Inserting deeply into the cancellous bone C. Inserting deeply into the cartilage D. Inserting deeply into the diaphysis E. Inserting deeply into the marrow

The correct answer is A. Fascial straps (retinacula) and fascial coverings of muscles or muscle groups characteristically attach to nearby bones by blending with the covering periosteum. No deep attachments are usually made by fascia. Cancellous bone (choice B) is spongy bone, which is usually found in marrow and is not the site for fascial attachment. Fascia do not usually attach to cartilage (choice C). Fascia attaches to bony shafts, or diaphyses (choice D), superficially via the periosteum.

Fascia do not penetrate the bone to reach the marrow (choice E).

A newborn baby is noted to have a left unilateral cleft lip. There are no abnormalities of the baby's palate. Which of the following developmental defects accounts for this occurrence? A. Failure of the left lateral palatine process to fuse with the median palatine process B. Failure of the left maxillary prominence to unite with the left medial nasal prominence C. Failure of the primary palate to fuse with the secondary palate D. Failure of the right and left medial nasal prominences to merge E. Failure of the right palatine process to fuse with the left palatine process

The correct answer is B. In the formation of the upper lip, the right and left medial nasal prominences merge to form the philtrum of the upper lip. The lateral maxillary prominence then merges with the merged medial nasal prominences. Failure of this merger to occur results in a unilateral cleft lip. Failure of a lateral palatine process to fuse with the median palatine process (choice A), which is synonymous with a failure of the primary palate to fuse with the secondary palate (choice C), results in a primary cleft palate. Primary clefts of the palate are found anterior to the incisive foramen of the palate. Primary cleft palates may occur along with cleft lips but are the result of a separate developmental defect. Failure of the right and left medial nasal prominences to merge (choice D) results in a median cleft lip. This is a rare anomaly. Normally, the right and left medial nasal prominences merge into a single prominence that forms the philtrum of the lip. Failure of the right and left palatine processes to fuse (choice E) results in a secondary cleft palate. Secondary cleft palates are found posterior to the incisive foramen. Normally, the right and left palatine processes fuse together and fuse to the primary palate.

A patient experiences visual difficulties. When a light is shined in her right eye, there is no pupillary response in either eye. However, upon shining a light in her left eye, both ipsilateral and contralateral pupillary responses are apparent. Her extraocular movements are intact. What is the most likely location of her lesion?

A. Oculomotor nerve, left side B. Oculomotor nerve, right side C. Optic nerve, left side D. Optic nerve, right side E. Trochlear nerve, left side

The correct answer is D. Know your cranial nerves! This woman has a "Marcus-Gunn pupil" with a defect in the afferent pathway of the optic nerve (in this case, on the right side). Recall that the afferent limb of the pupillary light reflex is the optic nerve (CN II); the efferent limb is the oculomotor nerve (CN III; parasympathetic fibers). When light is shined into her right eye, because her right optic nerve is not functioning properly, the light signal is not transmitted to the central nervous system (CNS), resulting in no pupillary response. As light is shined into her left eye, the left optic nerve transmits the signal to the CNS, which then sends an outbound signal through both the right and left oculomotor nerves to cause pupillary constriction in both eyes. Anatomically, the optic nerve is a special sensory nerve that originates in the retina of the eye, passes through the optic foramen of the sphenoid bone, and has a destination in the diencephalon via the optic chiasm. The oculomotor nerve (choices A and B) innervates all extraocular muscles except the lateral rectus (innervated by the abducens nerve) and the superior oblique (innervated by the trochlear nerve choice E). The oculomotor nerve also mediates pupillary constriction (parasympathetic fibers), eyelid opening (levator palpebrae), and innervates the ciliary muscle (allowing accommodation). A patient with a lesion of the left optic nerve (choice C) would have no pupillary responses in either eye when shining a light in the left eye; pupillary responses would be present in both eyes when shining a light in the right eye.

If the tongue deviates to the right side when protruded, the most likely cause is paralysis of which of the following muscles? A. Left genioglossus B. Left hyoglossus C. Left palatoglossus D. Right genioglossus E. Right hyoglossus

The correct answer is D. The genioglossus muscle is innervated by the hypoglossal nerve. The function of the genioglossus muscle is to pull the tongue forward (protrude) and toward the opposite side. When the right genioglossus muscle is paralyzed, the left genioglossus muscle pulls the tongue forward and to the right. If the left genioglossus muscle were paralyzed (choice A), the tongue would deviate toward the left on protrusion because of the unopposed action of the right genioglossus muscle. The left genioglossus muscle is innervated by the left hypoglossal nerve. The hyoglossus muscles (choices B and E) are innervated by the hypoglossal nerves. The function of these muscles is to retract the tongue. These muscles are not active during protrusion of the tongue. The palatoglossus muscle (choice C) are innervated by the vagus nerves rather than the hypoglossal nerves. Their function is to pull the tongue back (retract) and upward toward the palate.

Which of the following is a derivative of the second pharyngeal arch? A. Eustachian tube B. External auditory meatus C. Palatine tonsil D. Stylohyoid muscle E. Tensor tympani

The correct answer is D. The stylohyoid is derived from the second pharyngeal arch, which also gives rise to the muscles of facial expression, the stapedius, the posterior belly of the digastric muscle, Reichert's cartilage, and the facial nerve. The Eustachian tubes (choice A) are derived from the first pharyngeal pouch, which also gives rise to the middle ear cavity and the inner epithelial lining of the tympanic membrane. The external auditory meatus (choice B is derived from the first pharyngeal cleft, which also gives rise to the outer epithelial lining of the tympanic membrane.

The palatine tonsil (choice C) is derived from the epithelial lining of the second pharyngeal pouch. The tensor tympani (choice E) is derived from the first pharyngeal arch, which also gives rise to the muscles of mastication, the anterior belly of the digastric muscle, the mylohyoideus, the tensor veli palantini, the maxillary and mandibular bones, and the maxillary and mandibular divisions of the trigeminal nerve. Note that knowing the embryology of these structures helps you remember the innervation of the muscles of the face. First pharyngeal arch muscles of mastication trigeminal nerve Second pharyngeal arch muscles of facial expression facial nerve

Damage to the parasagittal region and falx cerebri will most likely result in which of the following neurologic deficits? A. Altered taste B. Leg paralysis C. Loss of facial sensation D. Ptosis E. Unilateral deafness

The correct answer is B. A meningioma of the parasagittal region and the falx cerebri would be located at the top of the brain, near the midline. In this position, it could compress the sensory or motor cortex supplying the lower extremities. The falx cerebri is a fold of dura mater that projects between the cerebral hemispheres in the longitudinal tissues. Its interior portions attach anteriorly to the crista galli and posteriorly to the internal occipital crest. Taste (choice A) is supplied by cranial nerves VII, IX, and X. These nerves arise from the brainstem. Facial sensation (choice C) is supplied by cranial nerve V, the nuclei of which are in the brainstem. Furthermore, the area of the sensory cortex that subserves the face is on the lateral aspect of the cortex and would not be affected by a tumor in the parasagittal region.

Ptosis (choice D) can be caused by a deficit in cranial nerve III, which arises from the brainstem. Unilateral deafness (choice E) suggests damage to cranial nerve VIII, which arises from the brainstem.

In which of the following structures are the opacifications located with cataract formation? A. Aqueous humor B. Cornea C. Lens D. Optic nerve E. Retina

The correct answer is C. Cataracts are lens opacifications. It is not known whether senile cataracts represent disease or normal opacification with age. Cataracts may occur as a consequence of diabetes mellitus, long-term steroids, or congenital infections. They are successfully treated at present with lens extraction and implantation of prosthetic lenses. The diagnostic characteristics for cataracts are as follows: 1) blurred vision that is progressive over months to years, 2) no pain or redness is seen, and 3) lens opacities may be invisible or grossly visible. Aqueous humor (choice A) is continually replaced due to active secretion by the ciliary body. As such, it does not undergo opacification; it is in constant flux. Corneal opacification (choice B) is generally a consequence of squamous metaplasia, in which the transparent, nonkeratinized, squamous cells are replaced by opaque, keratinized, squamous cells. Squamous metaplasia is a reparative process, usually due to friction injury to the cornea or a vitamin A deficiency. The optic nerve (choice D) is not transparent, and it does not undergo opacification injury. It may, however, atrophy due to ischemic, traumatic, infective, or metabolic insults. The retina (choice E) consists of multiple layers of neural cells. The retina is transparent but is not the site of cataract formation.

A sharp instrument passing through the superior orbital fissure would most likely sever which of the following structures? A. Abducens nerve B. Facial nerve C. Mandibular nerve D. Maxillary nerve E. Middle meningeal artery

The correct answer is A. A good way to remember what passes through the superior oribital fissure is that everything that innervates the eye, other than the optic nerve, passes through this fissure. This incudes the oculomotor nerve(CN III), the trochlear nerve (CN IV), the ophthalmic nerve (V1), and the abducens nerve (CN VI). The facial nerve (CN VII; choice B) passes through the internal auditory meatus. The mandibular nerve (V3; choice C) passes through the foramen ovale. The maxillary nerve (V2; choice D) passes through the foramen rotundum. The middle meningeal artery (choice E) passes through the foramen spinosum.

A knife wound to the neck damages the posterior cord of the brachial plexus. Which of the following muscles would be most likely be paralyzed? A. Deltoid B. Flexor carpi ulnaris C. Flexor digitorum superficialis D. Flexor pollicis brevis E. Palmaris longus

The correct answer is A. The posterior cord supplies the axillary and radial nerves. Of the muscles listed, only the deltoid is supplied by one of these two nerves, specifically the axillary nerve. The deltoid originates from the clavicle and scapula. It inserts into the deltoid tuberosity of the humerus. This muscle is responsible for abduction of the arm. The flexor carpi ulnaris (choice B) is supplied by the ulnar nerve. The flexor digitorum superficialis (choice C), the flexor pollicis brevis (choice D), and the palmaris longus (choice E) are supplied by the median nerve.

During the process of meiosis, a single homologous chromosome pair fails to separate during the first meiotic division. This failure would be most likely to produce which of the following conditions if fertilization occurs and an embryo later develops? A. Balanced translocation B. Triploidy C. Trisomy D. Unbalanced translocation E. Uniploidy

The correct answer is C. Meiosis is cell division that produces gametes with half of the normal somatic chromosome complement. The process described is nondisjunction, which will cause one daughter cell to have 24 chromosomes, while the other will have 22 chromosomes. When a gamete with the normal 23 chromosomes combines at fertilization with a gamete with 22 or 24 chromosomes, the embryo will have 47 chromosomes (trisomy) or 45 chromosomes (monosomy). Nondisjunction can occur in either the first or second meiotic division. Balanced translocation (choice A) occurs when non-homologous chromosomes exchange genetic material in such a way that no critical genetic material is lost. Triploidy (choice B) is the term used when a cell has 69 chromosomes (3N or 3 sets), and can occur in tumors or when an egg is fertilized by two sperm. An unbalanced translocation (choice D) occurs when non-homologous chromosomes exchange genetic material with a net loss or gain of critical genetic material.

Uniploidy (choice E) is the state of having 23 chromosomes, seen normally in sperm and eggs.

The superior ophthalmic vein directly communicates with which of the following dural venous sinuses? A. Cavernous sinus B. Occipital sinus C. Sigmoid sinus D. Superior petrosal sinus E. Straight sinus

The correct answer is A. The anterior continuation of the cavernous sinus, the superior ophthalmic vein, passes through the superior orbital fissure to enter the orbit. Veins of the face communicate with the superior ophthalmic vein. Because of the absence of valves in emissary veins, venous flow may occur in either direction. Cutaneous infections may be carried into the cavernous sinus and result in a cavernous sinus infection, which may lead to an infected cavernous sinus thrombosis. The cavernous sinus is lateral to the pituitary gland and contains portions of cranial nerves III, IV, V1, V2, and VI, and the internal carotid artery. The occipital sinus (choice B) is at the base of the falx cerebelli in the posterior cranial fossa. It drains into the confluence of sinuses. The sigmoid sinus (choice C) is the anterior continuation of the transverse sinus in the middle cranial fossa. The sigmoid sinus passes through the jugular foramen and drains into the internal jugular vein. The superior petrosal sinus (choice D) is at the apex of the petrous portion of the temporal bone and is a posterior continuation of the cavernous sinus. The superior petrosal sinus connects the cavernous sinus with the sigmoid sinus. The straight sinus (choice E) is at the intersection of the falx cerebri and the falx cerebelli in the posterior cranial fossa. The straight sinus connects the inferior sagittal sinus with the confluence of sinuses.

In preparation for a procedure to remove the fingernail on an index finger, the physician would most likely anesthetize a branch of the A. anterior interosseus nerve B. median nerve C. musculocutaneous nerve D. radial nerve E. ulnar nerve

The correct answer is B. The median nerve supplies the surface of the lateral palm, the palmar surface of the first three digits, and the distal dorsal surface of the index and middle fingers (including the nail beds). Therefore, prior to performing surgery in this area, it is essential to anesthetize a branch of this nerve (possibly a proper digital branch) to eliminate pain sensation around the nail bed of the index finger. The median nerve of the branchial plexus distributes to the flexor muscles on the forearm (flexor carpi radialis and palmaris longus), the pronators (p. quadratus and p. teres), digital flexors, and skin over the lateral surface of the hand. Neither the anterior interosseus (choice A) nor the musculocutaneous (choice C) nerves supplies the hand. The anterior interosseous nerve supplies the flexor pollicis longus, the lateral half of flexor digitorum profundus, and pronator quadratus. The musculocutaneous nerve supplies the coracobrachialis, biceps, and most of the brachialis muscle, then becomes the lateral cutaneous nerve of the forearm. The radial nerve (choice D) supplies skin on the radial side of the dorsal surface of the hand, but not the fingertips. The ulnar nerve (choice E) supplies the palmar and dorsal surfaces of the medial hand, including the palmar and dorsal surfaces of the fourth and fifth digits.

Which glandular area secretes hormones which are products of tyrosine metabolism? A. Alpha cells of pancreas B. Beta cells of pancreas C. Adrenal cortex D. Adrenal medulla

E. Testes

The correct answer is D.The products of the adrenal medulla are epinephrine (adrenalin) and norepinephrine (noradrenalin). The pathway of production of these compunds is a s follows: tyrosine to DOPA to dopamine to norepinephrine to epinephrine. The hormone secreted by alpha cells of the pancreas is glucagon, while beta cells secrete insulin. Both are peptide hormones. The adrenal cortex secretes a variety of hormones including cortisol and aldosterone, both steroid hormones. The testes secrete testosterone, also a steroid hormone.

A newborn infant has some of its abdominal viscera protruding through a defect in the abdominal wall. Which of the following is the likely cause of this defect? A. Failure of the intestinal loop to retract from the umbilical cord B. Failure of the yolk stalk to degenerate C. Failure of peritoneal fusion D. Incomplete fusion of the lateral body folds E. Umbilical herniation

The correct answer is D. During the fourth week of development, the lateral body folds move ventrally and fuse in the midline to form the anterior body wall. Incomplete fusion results in a defect that allows abdominal viscera to protrude from the abdominal cavity, a condition known as gastroschisis. During development, the midgut normally herniates into the umbilical cord and then subsequently retracts into the abdominal cavity. Failure of the intestinal loop to retract from the umbilical cord (choice A) results in omphalocele. Failure of the yolk stalk to degenerate (choice B) results in an ileal (Meckel's) diverticulum or a vitelline fistula or cyst. In the early embryo, the gut tube is connected to the yolk sac by a narrow connection known as the yolk stalk. Normally, this connection degenerates. During development, certain peritoneal organs fuse with the posterior abdominal wall to become secondarily retroperitoneal. Failure of this peritoneal fusion (choice C) will result in certain organs that are normally immobile being mobile (e.g., mobile cecum).

Umbilical herniation (choice E) results from abdominal viscera protruding through a weakness in the abdominal wall after development. Such protrusions are covered by subcutaneous fascia and skin, distinguishing them from gastroschisis.

If a patient has a drooping right eyelid and a dilated right pupil, which of the following neural structures is most likely affected? A. Cervical sympathetic chain B. Facial nerve C. Oculomotor nerve D. Superior cervical ganglion E. Trigeminal nerve

The correct answer is C. The oculomotor nerve innervates the levator palpebrae superioris, which elevates the eyelid. This nerve also innervates the inferior oblique muscles, as well as the superior, inferior, and medial rectus muscles. The oculomotor nerve also contains preganglionic parasympathetic fibers that synapse, in the ciliary ganglion, on postganglionic parasympathetic nerve fibers that innervate the sphincter pupillae muscle, which constricts the pupil. A lesion of the oculomotor nerve may therefore result in both drooping of the eyelid (ptosis) and dilation of the pupil (mydriasis). The cervical sympathetic chain (choice A) contains preganglionic sympathetic nerve fibers, arising from the upper thoracic spinal cord, which ascend to the cervical sympathetic ganglia. A lesion of these nerves may result in Horner's syndrome, which includes a ptosis and miosis (pupillary constriction) and, often, anhidrosis (lack of sweating). The facial nerve (choice B) innervates the muscles of facial expression, including the orbicularis oculi muscle. A lesion of this nerve may therefore result in the inability to close the eye. The superior cervical ganglion (choice D) contains the cell bodies of postganglionic sympathetic nerves that innervate structures in the head. A lesion of this structure will cause Horner's syndrome. The trigeminal nerve (choice E) provides sensory innervation to much of the head. A lesion of this nerve may interfere with the corneal blink reflex.

An injection to anesthetize pain from a fracture of the seventh rib should be gien in what area? A. Seventh intercostal space immediately below the seventh rib in the midclavicular line Seventh intercostal space immediately below the seventh rib just lateral to the angle of the rib C. Seventh intercostal space immediately below the seventh rib just medial to the angle of the rib D. Sixth intercostal space immediately above the seventh rib in the midclavicular line B. E. Sixth intercostal space immediately above the seventh rib just lateral to the angle of the rib

The correct answer is B. The seventh intercostal nerve (the anterior ramus of the seventh thoracic spinal nerve) innervates the seventh rib. After passing through the intervertebral foramen between the seventh and eighth thoracic vertebrae, the nerve lies in the seventh intercostal space. After passing the angle of the rib, it occupies a position along the lower border of the rib, in the costal groove. Use of a local anesthetic at this point will anesthetize the rib. By the time the intercostal nerve has reached the midclavicular line (choice A), it has already innervated most of the rib. Use of an anesthetic at this point would not be effective. While the intercostal nerve is in the intercostal space medial to the angle of the rib (choice C), it is not along the lower border of the rib. The intercostal nerve does not lie along the upper border of the rib (choices D and E). Thus, injection at these sites would not be effective.

The nucleus that lies immediately medial (and deep) to the uncus is the A. amygdala B. caudate nucleus C. claustrum D. hippocampus E. putamen

The correct answer is A. The uncus, which is the medial protrusion of the parahippocampal gyrus, is an external structure seen on the ventral surface of the temporal lobe. The amygdala is a collection of nuclei that lies directly beneath the uncus. The caudate nucleus (choice B) is a deep nuclear structure that lies lateral to the lateral ventricles. The claustrum (choice C) is a thin and elongated nucleus that lies just medial to the insular cortex. The hippocampus (choice D) is a nuclear structure that lies in the interior of the parahippocampal gyrus. The putamen (choice E) is a nuclear structure that resides lateral to the caudate and medial to the claustrum.

A nursing home patient who aspirates while lying on his back would be most likely to develop pneumonia involving which of the following sites? A. Anterior segment of the right upper lobe B. Apical segment of the right lower lobe C. Inferior lingular segment of the left upper lobe D. Lateral segment of the right middle lobe E. Superior lingular segment of the left upper lobe

The correct answer is B. Aspiration pneumonia is a common complication observed in nursing home patients. The most probable site of the pneumonia can be anticipated by knowing the anatomy of the bronchial tree because the aspirated fluid usually flows downhill. In a supine or nearly supine patient, the fluid flows into the trachea and then into either of the (typically the right) main bronchi. The first posteriorly located branch is the one leading to the apical aspect of (either) lower lobe. The lateral and posterior segments of the lower lobes are also supplied by posteriorly branching segmental bronchi. In contrast, the posterior aspects of the upper lobes are somewhat protected by an initial anteriorly directed bifurcation before their segmental bronchi arise. All other segments of the bronchial tree and their corresponding portions of lung are more

anterior.

The hormone most responsible for regulating sodium balance is secreted from: A. Zona glomerulosa of the adrenal cortex B. Zona reticularis of the adrenal medulla C. Zona fasiculata of the adrenal medulla D. Zona fasiculata of the adrenal cortex E. Zona glomerulosa of the adrenal medulla

The correct answer is choice A.Firstly, the hormone involved is aldosterone, which acts to increase sodium resorption in the kidney. Note that aldosterone (a mineralcorticoid) and the glucocorticoids (cortisol, cortisone) are produced by the adrenal cortex, not medulla. The medulla, with a different developmental origin and cell type, produces catecholamines such as epinephrine and norepinephrine. The cortex, which is outside the medulla, is in three regions. On the outside is the Zona Glomerulosa, source of the mineralcorticoids. Inside of that is the Zona Fasiculata, which together with the innermost layer of the cortex, the Zona Reticularis, produce glucocortoids. Remember that interior to the Zona Reticularis, you will find the adrenal medulla. Also note that as a memory aid, the cortex layers from the outside in are G-F-R, like the GFR of the kidney.

During embryological development, hematopoiesis occurs in different organs at different times. Which of the following are the correct organs, in the correct sequence, at which hematopoiesis occurs embryologically? A. Amnion, yolk sac, placenta, bone marrow B. Placenta, liver and spleen, yolk sac, bone marrow C. Placenta, spleen and lymphatic organs, bone marrow D. Yolk sac, bone marrow, liver and spleen E. Yolk sac, liver, spleen and lymphatic organs, bone marrow

The correct answer is E. By the third week of development, hematopoiesis begins in the blood islands of the yolk sac. Beginning at 1 month of age and continuing until 7 months of age, blood elements are also formed in the liver. Hematopoiesis occurs in the spleen and lymphatic organs between 2 and 4 months, and in the bone marrow after 4 months.

A woman suffers a fracture of the left tenth and eleventh ribs. Which of the following organs is most likely to have been injured by these fractured ribs? A. Descending colon B. Jejunum C. Left adrenal gland D. Left kidney E. Spleen

The correct answer is E. The spleen is a soft, friable organ with a thin capsule and is subject to injury upon trauma to the left side of the abdomen. It is located in the upper left quadrant of the abdomen, deep to the left ninth, tenth, and eleventh ribs. It is the most commonly injured organ in the abdomen. The adult spleen contains the largest number of lymphoid tissues in the human body. The descending colon (choice A) lies in a retroperitoneal position on the left side of the posterior abdominal wall. The descending colon begins at the splenic flexure immediately inferior to the spleen. The jejunum (choice B) is a peritoneal structure suspended by a long mesentery. It is located primarily in the upper left quadrant of the abdomen. Its long mesentery allows the jejunum to be highly mobile and thus is not likely to be injured by trauma to the body wall. The left adrenal gland (choice C) is a retroperitoneal structure that lies near the upper pole of the left kidney. It is embedded within fat and is thus well protected from injury. The left kidney (choice D) is a retroperitoneal structure that is well protected by fat.

The tongue will move in which direction when protuded with surgical damage to the right hypoglossal nerve? A. Downward B. Upward C. Directly forward

D. To the right E. To the left

The correct answer is D. There are two ways to answer this question. The formula method is that a tongue with muscle or nerve injury will protrude toward the side of injury, in this case the right side. This is similar to the case of the mandible protruding toward the side of injury when a lateral pterygoid is injured.The logical method is to imagine intrinsic muscles and extrinsic protruders of the tongue on the right side not receiving stimulation from innervation. In this case, only the left side protruders will operate. The tongue will protrude only on the left side, with the immobile right side acting as a stationary pivot while the tongue moves from left to right (toward the injured side).

You are asked to hold your upper arm against your lateral chest wall, with the palm upward. You then rotate the hand so that the palm faces downward, without bending the wrist. This motion is known as: A. abduction of the forearm B. adduction of the forearm C. flexion of the forearm D. pronation of the forearm E. supination of the forearm

The correct answer is D. When the forearm is rotated from anatomic position (palms facing forward, thumbs out) so that the palm faces posteriorly, the forearm is said to be pronated. Abduction (choice A) raises the arm to a horizontal position away from the body; adduction

(choice B) is the reverse. Flexion (choice C) brings the arm or forearm forward, in front of the plane of the body. Rotation of the forearm so that the palm faces forward (i.e., into anatomic position) is referred to as supination (choice E).

An otherwise healthy student taking no medications is concerned because he has noticed several painless uniform "large bumps" at the back of his tongue. These are most likely A. aphthous ulcers B. candidal colonies C. circumvallate papillae D. filiform papillae E. fungiform papillae

The correct answer is C. The large bumps at the back of his tongue are circumvallate papillae. These are large circular structures surrounded by moat-like depressions. The lateral surfaces of these papillae contain taste buds. There are also small serous-only salivary glands in these papillae. Aphthous ulcers (choice A) are small, white, or red mouth lesions. Candidal colonies (choice B) appear in thrush, which occurs more commonly in the immunocompromised host or in those taking antibacterial drugs. You are told that the patient is healthy and not taking medications, making this condition unlikely. Filiform papillae (choice D) are the most numerous papillae of the tongue. They are small, elongated cones that create the tongue's rough texture. They do not contain taste buds. Fungiform papillae (choice E) are mushroom-shaped structures scattered among the filiform papillae. They frequently contain taste buds. They are intermediate in size between filiform and circumvallate papillae.

Attempts to straighten out a flexed thigh cause great pain in a patient with appendicitis. This is due to the position of the appendix near which muscle? A. Adductor magnus B. Biceps femoris C. Gluteus maximus D. Gracilis E. Psoas major

The correct answer is E. The path of the psoas major lies in the retroperitoneum and comes close to the appendix. Acute appendicitis can cause either infection or a sympathetic inflammation of the psoas. This produces clinically a "positive psoas sign," in which attempts to straighten the patient's flexed (to relieve pain) hip produce sometimes marked exacerbation of the pain. None of the other muscles listed pass near the appendix.

Which of the following structures does the fetal allantoic duct become in the adult? A. Cloaca B. Medial umbilical ligament C. Urachus D. Ureter E. Urethra

The correct answer is C. The urachus is a fibrous remnant that extends from the umbilicus to the urinary bladder. It is also known as the median umbilical ligament of the anterior abdominal wall. The cloaca (choice A) is the primitive, endoderm-lined region that receives the terminal portion of the hindgut. It is later subdivided into urogenital and anal areas. The medial umbilical ligament (choice B) is a paired structure located deep to the peritoneum of

the anterior abdominal wall. It is formed by the obliterated umbilical artery. The ureter (choice D) is the muscular tube that conveys urine from the kidney to the urinary bladder. The urethra (choice E) is the passageway that carries urine from the bladder to the perineum.

The smooth part of the right atrium derives from which of the following embryonic structures? A. Bulbus cordis B. Primitive atrium C. Primitive ventricle D. Sinus venosus E. Truncus arteriosus

The correct answer is D. The smooth part of the right atrium (the sinus venarum) is derived from the sinus venosus. The coronary sinus and the oblique vein of the left atrium also derive from the sinus venosus. The bulbus cordis (choice A) gives rise to the smooth part of the right ventricle (conus arteriosus) and the smooth part of the left ventricle (aortic vestibule). The primitive atrium (choice B) gives rise to the trabeculated part of the right and left atria. The primitive ventricle (choice C) gives rise to the trabeculated part of the right and left ventricles. The truncus arteriosus (choice E) gives rise to the proximal part of the aorta and the proximal part of the pulmonary artery.

Microscopic examination of a PAS-stained histological section of a Graafian follicle demonstrates a bright reddish-pink, acellular ring around the ovum. Which of the following terms most accurately describes this ring?

A. Corona radiata B. Cumulus oophorus C. Theca externa D. Theca interna E. Zona pellucida

The correct answer is E. The ring described is the zona pellucida, which surrounds the ovum. The zona pellucida is rich in polysaccharides and glycoproteins and consequently stains brightly pink or red with PAS stain. Binding of the sperm cell membrane to the zona pellucida triggers the acrosome reaction, during which acrosomal enzymes are released that digest the zona pellucida, allowing the spermatozoon to contact and fuse with the ovum cell membrane. Anatomically, as layers of granulosa cells develop around the primary oocyte, microvilli from the surrounding granulosa cells intermingle with cells of the primary oocyte. The microvilli are surrounded by a layer of glycoproteins, and the entire region is called the zona pellucida. The follicular cells immediately outside the zona pellucida form the corona radiata (choice A). The larger cumulus oophorus (choice B) is the hill of follicular cells that surrounds the ovum. The theca interna (choice D) and externa (choice C) are formed from the connective tissue surrounding the follicle.

Following a surgical procedure on the right side of the neck, a patient can no longer raise his right arm above the horizontal position. The patient also cannot shrug his right shoulder. Which of the following nerves was injured? A. Axillary nerve B. Great auricular nerve C. Greater occipital nerve D. Spinal accessory nerve E. Transverse cervical nerve

The correct answer is D. The spinal accessory nerve crosses the posterior triangle of the neck

immediately deep to the investing fascia of the neck. This nerve innervates the trapezius muscle, which is responsible for upward rotation and elevation of the scapula. A lesion of this nerve in the posterior triangle leads to paralysis of the trapezius. Without the ability to upwardly rotate the scapula, abduction and flexion of the arm above the horizontal plane is not possible. Also, shrugging of the shoulder is impaired with paralysis of the trapezius muscle. The axillary nerve (choice A) does not pass through the neck. It is a branch of the brachial plexus, and it leaves the axilla to innervate the deltoid and teres minor muscles. The great auricular nerve (choice B) and transverse cervical nerve (choice E) are branches of the cervical plexus, which provide cutaneous innervation to the skin of the neck. No muscles are innervated by these nerves. The greater occipital nerve (choice C) is the dorsal ramus of the second cervical spinal nerve. It provides cutaneous innervation to the skin of the back of the head. No muscles are innervated by this nerve.

Zygomycosis, a destructive fungal infection of the sinuses, is likely to reach the brain by which of the following routes? A. Cavernous sinus B. External carotid artery

C. Internal carotid artery D. Superior sagittal sinus E. Superior vena cava

The correct answer is A. This question requires knowledge of pathophysiology with a basic understanding of anatomy. The cavernous sinuses are located on either side of the body of the sphenoid bone and become a potential route of infection because they receive blood both from the face (via the ophthalmic veins and sphenoparietal sinus) and from some of the cerebral veins. The spread of infection into the cavernous sinus can produce either central nervous system (CNS) infection or cavernous sinus thrombosis, both of which are potentially fatal. The route from the face to the brain is not arterial (choices B and C). The superior sagittal sinus (choice D) is located in the falx cerebri and drains venous blood from

the brain to other dural sinuses, from which it eventually drains into the jugular vein. Zygomycosis does not reach the brain by way of the superior sagittal sinus. The superior vena cava (choice E) drains blood from the upper part of the body into the heart.

A surgeon inadvertently sections the recurrent laryngeal nerve during a procedure. Which of the following muscles would retain its innervation subsequent to this injury? A. Cricothyroid B. Lateral cricoarytenoid C. Posterior cricoarytenoid D. Thyroarytenoid E. Vocalis

The correct answer is A. The recurrent laryngeal nerve is a branch of the vagus nerve, which innervates all of the intrinsic laryngeal muscles, except for the cricothyroid muscle. The cricothyroid is attached to the cricoid cartilage and the thyroid cartilage; contraction of this muscle tends to stretch and adduct the vocal ligament. The cricothyroid is innervated by the external laryngeal nerve. The lateral cricoarytenoid muscle (choice B) is innervated by the recurrent laryngeal nerve and is attached to the cricoid cartilage and the arytenoid cartilage. Its contraction causes adduction of the vocal ligament. The posterior cricoarytenoid muscle (choice C) is innervated by the recurrent laryngeal nerve and is attached to the cricoid cartilage and the arytenoid cartilage. Its contraction causes abduction of the vocal ligament. The thyroarytenoid muscle (choice D) is innervated by the recurrent laryngeal nerve and is attached to the thyroid cartilage and the arytenoid cartilage. Its contraction causes slackening of the vocal ligament. The vocalis muscle (choice E) is the most medial part of the thyroarytenoid muscle. It attaches to either the thyroid cartilage and the vocal ligament, or the arytenoid cartilage and the vocal ligament. It is innervated by the recurrent laryngeal nerve. Its contraction causes tension on segments of the vocal ligament.

As a result of a viral infection, a patient has swelling of the left facial nerve within the facial canal. The patient's face appears asymmetrical, and he complains that saliva drips from his mouth while he is chewing. Paralysis of which of the following muscles accounts for these symptoms? A. Buccinator B. Masseter C. Palatoglossus D. Palatopharyngeus E. Temporalis

The correct answer is A. Compression of the facial nerve within the facial canal may result in facial palsy (Bell's palsy). Because the muscles on one side of the face are paralyzed, the face appears asymmetrical. The buccinator muscle, which is located within the cheek and is innervated by the facial nerve, functions to hold food against the teeth while it is being chewed. Paralysis of this muscle can result in food and saliva accumulating between the teeth and the cheek. The buccinator originates from the alveolar processes of the maxilla and mandible. It inserts into the fibers of the obicularis oris. The masseter and temporalis muscles (choices B and E) are innervated by the mandibular division of the trigeminal nerve. These muscles of mastication function to elevate the mandible. The palatoglossus (choice C) and palatopharyngeus (choice D) muscles are innervated by the vagus nerve. The palatoglossus, with its mucosal covering, forms the palatoglossal fold (anterior pillar of the fauces), immediately anterior to the palatine tonsil. This muscle functions to draw the tongue and soft palate closer together, as occurs during swallowing. The palatopharyngeus, with its mucosal covering, forms the palatopharyngeal fold (posterior pillar of the fauces), which is immediately posterior to the palatine tonsil. This muscle causes elevation of the pharynx, as occurs during swallowing.

Biopsy demonstrates epithelial metaplasia. Which of the following cell types was most likely observed in the involved areas? A. Ciliated columnar epithelium

B. Cuboidal epithelium C. Keratinizing squamous epithelium D. Nonciliated columnar epithelium E. Nonkeratinizing squamous epithelium

The correct answer is D. The medical condition is Barrett's esophagus, in which the normally nonkeratinizing squamous epithelium (choice E) of the esophagus undergoes metaplasia to gastric or intestinal-like epithelium composed of nonciliated columnar epithelial cells. Barrett's esophagus typically develops in the setting of chronic gastroesophageal reflux and significantly increases the risk of later development of adenocarcinoma of the distal esophagus. Ciliated columnar epithelium (choice A) is found in the respiratory tract. Cuboidal epithelium (choice B) is found in the kidney, peritoneal lining, and pleural lining. Keratinizing squamous epithelium (choice C) is found in skin. Nonkeratinizing squamous epithelium (choice E), in addition to being the normal epithelium of the esophagus, is found in mouth, nose, and vagina.

Despite blockage of the celiac trunk, the organs receiving their blood supply from the trunk continue to operate normally. This is due to anastomoses between which vessels? A. Left gastroepiploic artery and right gastroepiploic artery B. Left gastroepiploic artery and right gastroepiploic artery C. Proper hepatic artery and gastroduodenal artery D. Right colic artery and middle colic artery E. Superior pancreaticoduodenal artery and inferior pancreaticoduodenal artery

The correct answer is E. The superior pancreaticoduodenal artery is a branch of the gastroduodenal artery, which is a branch of the common hepatic artery, itself a branch of the celiac trunk. The inferior pancreaticoduodenal artery is a branch of the superior mesenteric artery. Occlusion of the celiac trunk would allow blood from the superior mesenteric artery to

reach the branches of the celiac trunk via the connections between the superior and inferior pancreaticoduodenal arteries. The left gastric and right gastric arteries (choice A) both receive their blood from the celiac trunk. The left gastric artery is a direct branch of the celiac trunk. The right gastric artery is usually a branch of the proper hepatic artery, which is a branch of the common hepatic artery (a branch of the celiac trunk). The left and right gastroepiploic arteries (choice B) both receive their blood supply from the celiac trunk. The left gastroepiploic artery is a branch of the splenic artery, which is a branch of the celiac trunk. The right gastroepiploic artery is a branch of the gastroduodenal artery, which is a branch of the common hepatic artery (a branch of the celiac trunk). The proper hepatic and gastroduodenal arteries (choice C) are branches of the common hepatic artery, which is a branch of the celiac trunk. The right colic and middle colic arteries (choice D) are both branches of the superior mesenteric artery.

Damage to which of the following structures might produce hair cell loss? A. Basilar membrane B. Organ of Corti C. Reissner's membrane D. Scala tympani E. Scala vestibuli

The correct answer is B. Hearing is the detection of sound, which consists of pressure waves conducted through air or water. The receptors of the cochlear duct provide us with a sense of hearing that allows us to detect the quietest whisper and yet remain functional in a crowded, noisy environment. The organ of Corti contains hair cells from the cochlear branch of the vestibulocochlear nerve (CN VIII). These cells rest on the basilar membrane (choice A), which separates the scala tympani (choice D) from the scala media. The hair cells are embedded in the tectorial membrane, and movement of the basilar membrane below the cells causes the hairs to bend, which generates action potentials. The tectorial membrane that lies on the hair cells does not form a boundary between the different scala; the membrane separating the scala media from

the scala vestibuli (choice E) is Reissner's (vestibular) membrane (choice C).

Which of the following locations in the embryo later forms the dorsal horn of the spinal cord? A. Alar plate B. Basal plate C. Neural crest D. Rostral end of neural tube E. Sulcus limitans

The correct answer is A. The spinal cord arises from the caudal end of the neural tube. During development, an alar and a basal plate is formed, separated by a longitudinal groove called the sulcus limitans (choice E). The alar plate forms the dorsal (posterior) part of the spinal cord and becomes the sensory or afferent portion of the cord. The basal plate (choiceB) is the ventral (anterior) part of the cord and becomes the motor, or efferent, portion of the spinal cord and therefore would contain anterior horn cells. The neural crest (choice C) develops into multipolar ganglion cells of autonomic ganglia, pseudounipolar cells of spinal and cranial nerve ganglia, leptomeningeal cells, Schwann cells, melanocytes, chromaffin cells of the adrenal medulla, and odontoblasts. The brain forms from the rostral end of the neural tube (choice D).

From which of the following fetal vessels do the umbilical arteries arise? A. Aorta B. Carotid arteries C. Ductus arteriosus D. Iliac arteries E. Pulmonary arteries

The correct answer is D. The paired umbilical arteries arise from the iliac arteries. They supply unoxygenated fetal blood to the placenta. The single umbilical vein takes the newly oxygenated fetal blood from the placenta to the liver and then to the inferior vena cava via the ductus venosus. Near the level of vertebra L4, the terminal segment of the abdominal aorta divides to form the right and left common iliac arteries. These arteries carry blood to the pelvis and lower limbs. As these arteries travel along the inner surface, they descend behind the cecum and sigmoid colon, where each divides to form the internal iliac artery and external iliac artery.

A mass in the anterior midline of the neck, slightly above the larynx is mobile and elevates upon protrusion of the tongue. This mass is most likely a cyst that developed from which of the following embryonic structures? A. First pharyngeal cleft B. First pharyngeal pouch C. Second pharyngeal cleft D. Second pharyngeal pouch E. Thyroglossal duct

The correct answer is E. The thyroglossal duct develops as an evagination of the floor of the pharynx in the region where the tongue develops. The adult foramen cecum of the tongue marks the site of this evagination. The distal end of this duct normally forms the thyroid gland; the proximal part of the duct normally degenerates. Failure of a part of the duct to degenerate may lead to a thyroglossal duct cyst or a median cervical cyst, as seen in this patient. The first pharyngeal cleft (choice A) forms the external ear canal. This cleft normally remains patent. The first pharyngeal pouch (choice B) forms the middle ear cavity and the auditory tube. This pouch normally remains patent. The second pharyngeal cleft (choice C) normally does not remain patent. It is typically covered over by the overgrowth of the second pharyngeal arch. If part of this pouch does remain patent, it may form a lateral cervical cyst, which is seen on the lateral side of the neck along the anterior

border of the sternocleidomastoid muscle. The second pharyngeal pouch (choice D) forms the tonsillar fossa of the pharynx. The pharyngeal mucosa in this area arises from the endoderm of the pouch. Ingrowth of mesoderm cells results in the formation of the palatine tonsil.

Which of the following sites contains striated muscle that is not under voluntary control? A. Bladder B. Colon C. Esophagus D. Gallbladder E. Stomach

The correct answer is C. Striated (skeletal) muscle not under voluntary control is an unusual feature of the upper and middle thirds of the esophagus. The middle third of the esophagus contains roughly half striated and half smooth muscle; the lower third contains only smooth muscle. All the other structures listed in the answer choices contain smooth muscle. As a side note, cardiac muscle cells do not rely on nerve activity to start a contraction. Instead, specialized pacemaker cells establish a regular rate of contraction. Because these pacemaker cells regulate the heart and the central nervous system does not, cardiac muscle is also considered striated, involuntary muscle.

An ulcer damages an artery supplying the area of the greater curvature of the stomach. Which artery is involved? A. Left gastric B. Left gastroepiploic C. Right gastric D. Right gastroepiploic

E. Short gastric

The correct answer is D. The right gastroepiploic artery, off the gastroduodenal artery, supplies the right half of the greater curvature of the stomach and could be directly affected by ulceration of the greater curvature of the stomach at a site this close (4 cm) to the pyloric sphincter. The left gastric artery (choice A), off the celiac trunk, supplies the left half of the lesser curvature of the stomach. The left gastroepiploic artery (choice B), off the splenic artery, supplies the left half of the greater curvature of the stomach. Although it anastomoses with the right gastroepiploic artery, it is unlikely that this artery would be directly damaged by ulceration of the stomach near the pyloric sphincter. The right gastric artery (choice C), off the proper hepatic artery, supplies the right half of the lesser curvature of the stomach. The short gastric artery (choice E), actually one of several (4 to 5) short gastric arteries, off the splenic artery (occasionally the left gastroepiploic), supplies the fundus of the stomach, which is the most distant from the pylorus.

An elderly patient has had multiple small strokes lending to an absence of the gag reflex. These findings suggest involvement of the nucleus of which of the following cranial nerves? A. Facial (VII) B. Glossopharyngeal (IX) C. Hypoglossal (XII) D. Spinal accessory (XI) E. Vestibulocochlear (VIII)

The correct answer is B. Cranial nerve IX is the glossopharyngeal nerve, which has a nucleus in the medulla and is necessary for the gag reflex. The gag reflex is elicited by touching either side of the posterior pharynx with a tongue blade, producing bilateral elevation of the palate and bilateral contraction of the pharyngeal muscles. The afferent of this reflex arc consists of the

ipsilateral glossopharyngeal nerve, while the vagus nerve, bilaterally, supplies the efferent limb. Although the glossopharyngeal nerve may seem to be one of the less important cranial nerves, you should remember to test for its function, as a loss of gag reflex can lead to the patient's death secondary to an aspiration pneumonia. The glossopharyngeal nerve is a mixed sensory and motor nerve to the head and neck. It originates from the posterior 1/3 of the tongue, pharynx, palate, and carotid arteries of the neck. Its destination includes sensory nuclei of the medulla oblongata, as well as the pharyngeal muscles involved in swallowing. Cranial nerve VII (choice A) is the facial nerve, which supplies motor function to the face, but it does not supply the oropharynx. Cranial nerve XII (choice C) is the hypoglossal nerve, which supplies the intrinsic and most extrinsic muscles of the tongue. It is not involved in the gag reflex. Cranial nerve XI (choice D) is the spinal accessory nerve, which supplies the trapezius and sternocleidomastoid. Cranial nerve VIII (choice E) is the vestibulocochlear nerve, responsible for hearing and equilibrium.

Which of the following respiratory system components is derived from neural crest? A. Endothelial cells B. Epithelium of primary bronchi C. Laryngeal cartilage D. Tracheal glands E. Type I pneumocytes

The correct answer is C. Laryngeal cartilages are derived from neural crest. The larynx is composed of three cartilages that form the "body of the larynx": the thyroid cartilage, the cricoid cartilage, and the epiglottis. The larynx also contains three pairs of smaller hyaline cartilages: the arytenoid, corniculate, and cuneiform cartilages. The endothelial cells (choice A), in the simple squamous epithelium that lines the pulmonary capillaries, are derived from visceral mesoderm.

The epithelial lining of primary bronchi (choice B) is derived from endoderm. Tracheal glands (choice D) and epithelium both derive from endoderm. Type I pneumocytes (choice E) are derived from endoderm.

The extraocular muscles are derived from which of the following structures? A. Branchial arches B. Optic cup ectoderm C. Somites D. Somitomeres E. Splanchnic mesoderm

The correct answer is D. The somitomeres are specialized masses of mesoderm found in the head region that give rise to the muscles of the head. The extraocular muscles are derived from somitomeres 1, 2, 3, and 5. The branchial arches (choice A) give rise to muscles of mastication (arch 1), muscles of facial expression (arch 2), and muscles of the pharynx and larynx (arches 3-6), as well as additional small muscles. The optic cup ectoderm (choice B) gives rise to the muscles of the iris (sphincter and dilator pupillae). These are the only muscles not formed from mesoderm. Somites (choice C) give rise to the inferior muscles of the neck. Splanchnic mesoderm (choice E) gives rise to smooth muscle of the viscera and the heart muscle.

A CT scan reveals a small tumor at the cerebellopontine angle of the brain. Which of the following nerves is most likely to be affected by this tumor?

A. Facial nerve B. Glossopharyngeal nerve C. Optic nerve D. Trigeminal nerve E. Vagus nerve

The correct answer is A. The facial nerve and the vestibulocochlear nerves emerge from the brain stem at the cerebellopontine angle. These are the two nerves that will be initially affected by a tumor in this region. The entire anatomy of the facial nerve is as follows: the facial nerve originates from the taste receptors on the anterior 2/3 of the tongue (sensory) and from nuclei of the pons (motor). It passes through the internal acoustic canal of the temporal bone to reach the stylomastoid foramen. The glossopharyngeal and vagus nerves (choices B and E) emerge from the brain stem at the postolivary sulcus. This is caudal to the cerebellopontine angle. The optic nerve (choice C) exits from the optic chiasm on the ventral surface of the diencephalon. This is rostral to the cerebellopontine angle. The trigeminal nerve (choice D) emerges from the brain stem at the anterolateral surface of the pons. This is rostral and ventral to the cerebellopontine angle.

A patient is unable to close her right eye. Physical examination reveals weakness of the right orbicularis oculi. Which of the following additional symptoms would likely also be present? A. Blurred vision B. Hyperacusis C. Inability to chew D. Inability to feel the face E. Inability to shrug the shoulder

The correct answer B. The facial nerve innervates the muscles of facial expression. The

visceral motor portion innervates the lacrimal gland and nasal mucous glands via the sphenopalatine ganglion. The submandibular and sublingual salivary glands are innervated via the submandiublar ganglion. This patient has a lesion of the facial nerve (VII), which leads to an inability to close the ipsilateral eye because of damaged motor fibers to the orbicularis oculi. This patient would also lose her corneal reflex on that side because of an inability to blink and would have ipsilateral paralysis of the muscles of facial expression distal to the lesion. If the lesion affected the facial nerve more proximally, additional findings would be hyperacusis (increased sensitivity to sound because of stapedius muscle paralysis), lack of taste sensation in the anterior two-thirds of the tongue, and disturbed lacrimation and salivation. Blurred vision (choice A) could occur with lesions of the oculomotor (CN III), abducens (CN VI), or trochlear (CN IV) nerves, which innervate the extraocular muscles. CN III innervates the medial rectus, inferior rectus, superior rectus, and inferior oblique muscles. CN VI innervates the lateral rectus and CN IV innervates the superior oblique. An inability to chew (choice C) would probably be the result of a lesion of the trigeminal nerve (CN V). Motor fibers of CN V innervate the muscles of mastication (temporalis, masseter, and medial and lateral pterygoid muscles), and a lesion of these fibers may cause the jaw to deviate to the side of the weak muscles. An inability to feel the face (choice D) would also be the result of a CN V lesion. This lesion could result in the ipsilateral loss of general sensation of the face and also of the mucous membranes of the oral and nasal cavities. A lesion of the accessory nerve (CN XI) would cause paralysis of the trapezius muscle, which results in a sagging of the shoulder and a weakness in attempting to shrug the shoulder (choice E).

A patient received a severe blow to the lateral side of the head, resulting in an epidural hematoma. Which of the following blood vessels was most likely torn? A. Anterior cerebral artery B. Middle cerebral artery C. Middle meningeal artery D. Superficial temporal artery E. Superior cerebral vein

The correct answer is C. The middle meningeal artery is in the interior of the lateral portion of the cranial cavity, embedded in the periosteal (outer) layer of the dura. A tear of this artery results in blood entering the potential space between the outer dural layer and the skull (epidural space), causing an epidural hematoma. The anterior cerebral artery and middle cerebral artery (choices A and B) lie on the surface of the brain. The anterior cerebral arteries supply the medial surface of the cerebral hemispheres, and the middle cerebral arteries supply the lateral surface of the cerebral hemispheres. A tear of either of these arteries would result in blood entering the subarachnoid space (subarachnoid hemorrhage). The superficial temporal artery (choice D) is a branch of the external carotid artery and is external to the skull. This artery supplies the skin and other tissue of the temple region. The superior cerebral veins (choice E) drain the cerebral hemisphere and enter the superior sagittal sinus. A tear of these veins results in blood entering the potential space between the dura and arachnoid (subdural space), causing a subdural hematoma.

The x-ray of a child's arm after a fall appears to show a fracture near, but not at, the distal end of the ulna. Before diagnosing a fracture, you should also consider the possibility that this is actually which of the following? A. Articular cartilage B. Epiphyseal plate C. Perichondrium D. Primary ossification center E. Secondary ossification center

The correct answer is B. The epiphyseal plate of the bone contains cartilage that is radiolucent. The plate in a bone that is not yet fully ossified can produce a "line" crossing the bone near the end. This may be easily mistaken for a fracture by the inexperienced. Anatomically, the epiphyseal plate separates the epiphysis from the diaphysis. Articular cartilage (choice A) is radiolucent, but occurs at the very tip of the long bones. Perichondrium (choice C) is usually difficult to see on x-ray.

Primary (choice D) and secondary (choice E) ossification centers are radiopaque.

Which of the following embryonic structures gives rise to the adrenal cortex? A. Ectoderm B. Endoderm C. Mesoderm D. Mesonephros E. Neural crest cells

The correct answer is C. The mesoderm gives rise to the adrenal cortex. In addition, it also gives rise to connective tissue, cartilage, bone, muscle, blood and lymph vessels, kidneys, gonads, serous membranes lining body cavities, and the spleen. The ectoderm (choice A) gives rise to the central nervous system, peripheral nervous system, epidermis and its appendages, mammary glands, pituitary gland, tooth enamel, and the neural crest. The endoderm (choice B) gives rise to the parenchyma of the tonsils, thyroid and parathyroid glands, thymus, liver, pancreas, the epithelial lining of the gastrointestinal and respiratory tracts, urinary bladder, urethra, and auditory tube. The mesonephros (choice D) functions as an interim kidney in the embryo. The neural crest cells (choice E) give rise to cells of the spinal and cranial nerves, autonomic ganglia, melanocytes, leptomeninges, connective tissue and bone of branchial arch origin, and the adrenal medulla.

The major structural component of the sperm flagellum is the: A. microtubule B. microfilament

C. actin filament D. mysosin filament E. ciliary body

The correct answer is choice A. This is a basic histological question of a type found commonly on NBDE, which asks about basic cellular organelles and structures. The sperm cell flagellum is notable for a 9+2 arrangement of fibers formed from microtubules. The 9 single fibers form a circular ring in cross section around the 2 doubled fibers in the center. One of the significant facts about the fibers is that they are composed of microtubules, made up of tubulin. The other significant fact is that all eucaryotic flagella and cilia have this same structure (for example, cilia form the tracheal lining). Note that procaryotic (bacterial) flagellea do NOT share this structure.

In cases of dysphagia where the esophagus is compressed, which structure would be most likely to cause the compression? A. Left atrium B. Left ventricle C. Pulmonary trunk

D. Right atrium E. Right ventricle

The correct answer is A. The left atrium forms most of the posterior wall of the heart. The esophagus passes immediately posterior to the heart. Enlargement of the left atrium may compress the esophagus and cause dysphagia. Anatomically, the esophagus begins posterior to the cricoid cartilage, at the level of vertebrum C6. From this point, it descends toward the thoracic cavity posterior to the trachea, passes inferiorly along the dorsal wall of the mediastinum, and enters the abdominopelvic cavity through an opening in the diaphragm, the diaphragmatic hiatus. The left ventricle (choice B) forms most of the left border of the heart and most of the diaphragmatic surface of the heart. The left ventricle is not related to the esophagus. The pulmonary trunk (choice C) emerges from the right ventricle on the anterior surface of the

heart. The pulmonary trunk is not related to the esophagus. The right atrium (choice D) forms the right border of the heart. It is not related to the esophagus. The right ventricle (choice E) forms most of the anterior wall of the heart and a small portion of the diaphragmatic surface of the heart. It is not related to the esophagus.

Which of the following tissues normally has the highest percentage of mucus-secreting cells? A. Esophageal mucosa B. Oral mucosa C. Parotid gland D. Sublingual gland E. Submandibular gland

The correct answer is D. The sublingual salivary glands are located beneath the mucous membrane of the floor of the mouth. Numerous sublingual ducts open along either side of the lingual frenulum. Salivary glands can contain predominantly serous cells, predominantly mucous cells, or both in their acini. As you proceed from the midline laterally, acini in the sublingual gland are almost pure mucous cells, whereas acini in the submandibular gland (choice E) contain a mixture of serous and mucous cells. Acini in the parotid gland (choice C) are mostly pure serous cells. The esophageal mucosa (choice A) and the oral mucosa (choice B) are squamous epithelia that do not form acini.

The primary arterial supply to the nasal mucosa is a direct branch of which of the following arteries? A. Facial artery B. Maxillary artery C. Superficial temporal artery

D. Superior labial artery E. Transverse facial artery

The correct answer is B. The major source of blood supply to the nasal mucosa is the sphenopalatine artery, which is the terminal branch of the maxillary artery. The sphenopalatine artery enters the nasal cavity from the pterygopalatine fossa by passing through the sphenopalatine foramen in the lateral wall of the nasal cavity. The facial artery (choice A) is a direct branch of the external carotid artery. It provides most of the blood supply to the superficial face. The superficial temporal artery (choice C) is a terminal branch of the external carotid artery. The other terminal branch is the maxillary artery. The superficial temporal artery provides blood supply to the temporal region and the lateral portion of the scalp. The superior labial artery (choice D) is a branch of the facial artery. The superior labial artery provides blood supply to the upper lip. It has a septal branch that provides some of the blood supply to the anterior portion of the septal mucosa of the nasal cavity. However, this is not the major blood supply to the nasal cavity. The transverse facial artery (choice E) is a branch of the superficial temporal artery. It provides blood supply to the parotid gland, the parotid duct, and the skin of the lateral face.

Which of the following cranial nerves carries the pain sensation from the tip of tongue? A. V2 B. V3 C. VII D. IX E. X

The correct answer is B. This question is essentially asking "which of the following nerves innervates the tip of the tongue?" The innervation of the tongue is complex. The mandibular division of the trigeminal nerve (V3) carries general somatic sensation from the anterior two-

thirds of the tongue. The mandibular branch is also a sensory nerve for the lower gingiva, teeth, and lips, as well as the palate. The maxillary division (V2, choice A) carries somatic sensation from the palate, upper gingiva, and upper lip, as well as the lower eyelid, cheek, nose, and a portion of the pharynx. The facial nerve (VII, choice C) carries taste from the anterior two-thirds of the tongue, as well as innervating the lacrimal gland and the submandibular and sublingual salivary glands. The glossopharyngeal nerve (IX, choice D) carries sensation and taste from the posterior onethird of the tongue. The vagus nerve (X, choice E) carries sensation from the lower pharynx.

A mass is noted at the back of a young man's tongue. A biopsy's pathology report comes back with a diagnosis of normal thyroid tissue. This finding is related to the embryonic origin of the thyroid near which of the following structures? A. First pharyngeal pouch B. Foramen cecum C. Nasolacrimal duct D. Second pharyngeal arch E. Third pharyngeal pouch

The correct answer is B. The thyroid gland originates as a mass of endodermal tissue near the foramen cecum, which is near the tuberculum impar (which becomes the central part of the tongue). During development, the thyroid descends in front of the pharynx, maintaining a connection to the tongue via the thyroglossal duct. Usually, the thyroglossal duct disappears. Uncommonly, residual ectopic thyroid tissue can be left anywhere along the path, including at the back of the tongue. (In rare cases, all of the thyroid tissue remains at this site, forming a mass that should not be excised, for obvious reasons!) Anatomically, the thyroid gland curves across the anterior surface of the trachea just below the thyroid cartilage that forms the majority of the anterior surface of the larynx. The two lobes of the thyroid gland are united by a slender connection, the isthmus.

The first pharyngeal pouch (choice A) develops into the middle ear and eustachian tube. The nasolacrimal ducts (choice C) connect the eyes to the oropharynx. The second pharyngeal arch (choice D) develops into many muscles of the face and the styloid process of the temporal bone. The third pharyngeal pouch (choice E) develops into the thymus and inferior parathyroid glands.

If the nerve that accompanies the superior laryngeal artery is damaged, which of the following functional losses will ensue? A. Loss of sensation in the laryngeal mucosa above the vocal folds B. Loss of sensation in the laryngeal mucosa below the vocal folds C. Loss of sensation in the pharyngeal mucosa D. Paralysis of the cricothyroid muscle E. Paralysis of the lateral cricoarytenoid muscle

The correct answer is A. The superior laryngeal artery is a branch of the superior thyroid artery. It enters the larynx by passing through the cricothyroid membrane. In this region, it is accompanied by the internal branch of the superior laryngeal nerve. This nerve provides sensory innervation to the laryngeal mucosa above the vocal folds. The laryngeal mucosa below the vocal folds (choice B) receives its sensory innervation from the recurrent laryngeal nerve. The pharyngeal mucosa (choice C) receives its sensory innervation from the glossopharyngeal nerve. The cricothyroid muscle (choice D) receives its motor innervation from the external branch of the superior laryngeal nerve. The lateral cricoarytenoid muscle (choice E) receives its motor innervation from the recurrent laryngeal nerve.

An atrial septal defect results from failure of the A. ostium primum to form within the septum primum B. ostium secundum to form within the septum primum C. septum primum to fuse with the endocardial cushions D. septum primum to fuse with the septum secundum E. septum secundum to fuse with the endocardial cushions

The correct answer is C. The septum primum (first interatrial septum) develops by growing from the cranial end of the embryonic atrium toward the endocardial cushions. The gap that exists between the two atria during this period is the ostium primum. As the septum primum continues its growth, the ostium primum gets smaller until it is closed when the septum primum completes its growth and completely fuses with the endocardial cushions. Failure of the septum primum to fuse completely with the endocardial cushions leaves a persistent ostium primum, known as a primum-type atrial septal defect. The ostium primum does not form within the septum primum (choice A). The ostium primum is the communication between the two atria that exists during the formation of the septum primum. That is, the ostium primum is the space within the developing atrium not yet occupied by the septum primum. The ostium secundum normally forms within the septum primum (choice B) before the ostium primum closes by fusion of the septum primum with the endocardial cushions. Failure of the ostium secundum to form would result in embryonic death because there would be no pathway for blood to pass from the right atrium to the left atrium when the ostium primum closes, thus depriving the embryo of oxygenated blood. Most of the septum primum normally disappears. The part that remains forms the valve of the foramen ovale. This part of the septum primum normally does not fuse with the septum secundum (choice D) during prenatal life. After birth, the valve of the foramen ovale is pushed against the septum secundum as a result of the increased pressure in the left atrium. This achieves functional closure of the foramen ovale. Fusion does not normally occur at this time; it usually occurs later in life in most people. In some people, however, complete fusion never occurs (probe patency). The septum secundum normally does not fuse with the endocardial cushions (choice E).

A CT scan of the head demonstrates a mass in the olfactory groove area. The axons likely compressed by this mass project to which of the following structures? A. Insula B. Nucleus ambiguus C. Postcentral gyrus D. Precentral gyrus E. Pyriform cortex

The correct answer is E. The olfactory nerves can be damaged by head injury, severe infection, and tumors (such as meningiomas) that may compress the olfactory bulbs. Unilateral damage is often not noticed by the patient, but bilateral damage (which surprisingly can also be missed if it develops slowly) may lead to occasional bizarre errors in odor identification. The olfactory nerves are technically the neurons on the mucosal side of the cribriform plate that send processes through the plate to synapse in the olfactory bulb, which in turn sends axons to the pyriform cortex (the primary olfactory cortex). The insula (choice A) is thought to contain the primary gustatory (taste) cortex. The nucleus ambiguus (choice B) provides the special visceral efferent fibers carried by the glossopharyngeal and vagus nerves. The postcentral gyrus (choice C) contains the primary sensory cortex. The precentral gyrus (choice D) contains the primary motor cortex.

A biopsy specimen demonstrates a ciliated columnar epithelium. From which of the following locations in the female genital tract was the biopsy obtained? A. Cervix B. Endometrium C. Fallopian tube D. Ovary E. Vagina

The correct answer is C.The fallopian tube is the only structure in the female genital tract with a ciliated columnar epithelium; the beating of the cilia helps move the egg into the uterus. This fact is also sometimes clinically helpful because dilated and deformed fallopian tubes can be microscopically distinguished from cystic ovarian tumors by the presence of the cilia. Each uterine tube is a hollow, muscular tube measuring approximately 13 cm in length. Each uterine tube is divided into three regions: the infundibulum, the ampulla, and the isthmus. The cervix (choice A) and vagina (choice E) are lined by squamous epithelium. The endometrium (choice B) is lined by columnar epithelium (although a few ciliated cells may be present). The covering of the ovary (choice D) is cuboidal epithelium, and cysts within the ovary can be lined by cuboidal or nonciliated columnar epithelium.

A person sits with his legs together and then opens his legs against lateral resistance. Which nerve controls this type of movement? A. deep peroneal nerve B. femoral nerve C. obturator nerve D. sciatic nerve E. superior gluteal nerve

The correct answer is E. The motion described is hip abduction. The superior gluteal nerve from roots L4-S1 is responsible for hip abduction. The deep peroneal nerve (choice A) controls dorsiflexion of the foot. If you see a patient on the NBDE with "foot drop," think of damage to the deep peroneal nerve. The femoral nerve (choice B) is responsible for knee extension. The obturator nerve (choice C) is responsible for hip adduction. The hip adduction machine would provide resistance to the medial aspects of the thighs, and the individual would push

his/her legs together against the machine's resistance. The sciatic nerve (choice D) controls knee flexion.

During anatomy lab, a dental student notes a fibrous band that runs on the visceral surface of the liver. It is attached on one end to the inferior vena cava and on the other end to the left branch of the portal vein. In the embryo, this structure corresponds to the A. ductus venosus B. ligamentum teres C. ligamentum venosum D. umbilical arteries E. umbilical vein

The correct answer is A. This question could have tricked you if you didn't catch the key words, "in the embryo." If you read the question too quickly and thought you were going to be asked to identify the structure described, you probably chose choice C (ligamentum venosum) because that is indeed the structure in question. In the embryo, however, this fibrous band is actually the ductus venosus. The ductus venosus is an embryonic vessel that allows blood to bypass the fetal liver; this prevents the depletion of oxygen and nutrient-rich blood in the hepatic sinusoids. The embryonic umbilical arteries (choice D) become the medial umbilical ligaments. The embryonic umbilical vein (choice E) actually becomes the fibrous ligamentum teres (choice B). The ligamentum teres is located in the free margin of the falciform ligament.

Damage to the upper fibers of the trapezius muscle would most likely impair which of the following movements? A. Depression of the medial end of the scapula B. Elevation of the acromion

C. Flexion of the neck D. Raising of the medial border of the scapula E. Turning of the face sideways

The correct answer is B. The trapezius is a large muscle arising in the posterior midline from the superior nuchal line of the skull, ligamentum nuchae, and spinous processes of all of the thoracic vertebrae. The upper fibers, which are potentially damaged by wounds to the posterior and lateral neck, insert on the crest of the spine of the scapula, the medial border of the acromion, and the lateral third of the clavicle. The upper fibers serve to elevate the acromion. The lower fibers of the trapezius depress the medial end of the scapula (choice A). The sternocleidomastoids are important in extension of the head at the atlanto-occipital joint and flexion of the cervical vertebral column (choice C). The rhomboids and levator scapulae are important in raising the medial border of the scapula (choice D). Contraction of a single sternocleidomastoid muscle is important in turning of the face sideways (choice E) in the contralateral direction.

A man's chest is compressed during a car accident, causing a posterior displacement of the clavicle at the sternoclavicular joint. Which of the following structures would be most at risk? A. Aorta B. Esophagus C. Heart D. Superior vena cava E. Trachea

The correct answer is E. The sternoclavicular joint is quite strong and dislocates only with difficulty. Dislocations, particularly posterior dislocations, are occasionally seen and may be dangerous because of impingement onto the trachea, causing respiratory difficulties. The trachea is a tough flexible tube with a diameter of approximately 2.5 cm and a length of approximately

11 cm. The trachea begins anterior to the vertebra C6 in a ligamentous attachment to the cricoid cartilage and ends in the mediastinum at the level of vertebra T5. At this point, it branches to form the right and left primary bronchi. Both the aorta (choice A) and the superior vena cava (choice D) are more posterior than the trachea and are therefore less vulnerable. The esophagus (choice B) is located behind the trachea and is less vulnerable. The heart (choice C) lies below the clavicle and would not be at particular risk.

Which of the following cell types is derived from neuroepithelial cells? A. Astrocytes B. Enterochromaffin cells C. Melanocytes D. Odontoblasts E. Schwann cells

The correct answer is A. Astrocytes and oligodendrocytes are both derived from glioblasts, which, in turn, are derived from neuroepithelial cells. Other neuroepithelial cell derivatives include neuroblasts and ependymal cells. The astrocytes are the largest and most numerous glial cells. These cells are responsible for maintaining the blood-brain barrier, creating a threedimensional framework for the central nervous system, performing repairs in damaged neural tissues, and controlling the interstitial environment. All the other choices are derived from neural crest cells. Other neural crest derivatives include the neurons of the parasympathetic and sympathetic ganglia (including the adrenal medulla), the dorsal root ganglia of the peripheral nervous system, the sensory ganglia of cranial nerves V, VII, IX, and X, and the leptomeninges (pia and arachnoid).

Following a fracture of the humerus, which of the following is responsible for producing the majority of the new bone that will reunite the two fragments?

A. Cancellous bone B. Cartilage C. Compact bone D. Marrow E. Periosteum

The correct answer is E. When the periosteum is torn during a fracture, it supplies cells that develop into osteoblasts and are the major producers of the new bone that reunites the two ends. Heterotopic ossification (bone formed outside the regular bone) can occur as a complication of fracture if some of the osteoblastic cells are misdirected into adjacent tissues. The periosteum assists in the attachment of the osteoblasts to surrounding tissues and to associated tendons and ligaments. This cellular layer functions in bone growth and participates in repair after an injury. Pre-existing cancellous bone (choice A) and compact bone (choice C) are not the major source of osteoblasts that form the new bone. Cartilage (choice B) and marrow (choice D) do not contribute to new bone formation after fracture.

Which membrane is in the way is you try to reach the lesser peritoneal sac and head of the pancreas after penetrating the greater peritoneal sac? A. Falciform ligament B. Gastrohepatic ligament C. Gastrosplenic ligament D. Hepatoduodenal ligament E. Splenorenal ligament

The correct answer is B. The gastrohepatic ligament is the part of the lesser omentum that separates the greater peritoneal sac from the right portion of the lesser peritoneal sac. This portion of the lesser omentum has no significant blood vessels within it and may be incised for

surgical access. The falciform ligament (choice A) is a mesenteric membrane between the liver and the anterior abdominal wall. This ligament is within the greater peritoneal sac and does not separate it from the lesser peritoneal sac. The gastrosplenic ligament (choice C) passes from the greater curvature of the stomach to the spleen. It separates the greater peritoneal sac from the left portion of the lesser peritoneal sac. Incision of this structure would be the appropriate surgical approach to gain access to the left side of the lesser peritoneal sac. The hepatoduodenal ligament (choice D) is part of the lesser omentum and separates the greater peritoneal sac from the right portion of the lesser peritoneal sac. It forms the anterior border of the epiploic foramen. However, the hepatoduodenal ligament contains the common bile duct, the proper hepatic artery, and the portal vein, and therefore may not be incised for surgical access. The splenorenal ligament (choice E) passes from the spleen to the parietal peritoneum on the anterior surface of the left kidney. It separates the greater peritoneal sac from the left portion of the lesser peritoneal sac. This ligament contains the splenic artery, splenic vein, and the tail of the pancreas, and therefore may not be incised to gain access to the lesser peritoneal sac.

Which of the following fibers provide the only output from the cerebellar cortex? A. Climbing B. Golgi cell C. Granule cell D. Mossy E. Purkinje

The correct answer is E. Two basic things about cerebellar circuitry that are well-worth knowing are that Purkinje cells of the cerebellar cortex project to the deep cerebellar nuclei and that these nuclei project out of the cerebellum. Purkinje cells are located in the second (Purkinje) cell layer of the cerebellar cortex and form inhibitory synapses on the deep cerebellar nuclei. Climbing fibers (choice A) are afferents to the cerebellum. Specifically, they originate from the medullary olivary nuclear complex, enter the cerebellum through the inferior cerebellar

peduncle, and project to the deep cerebellar nuclei and the lateral cerebellar hemispheres. Golgi cell bodies (choice B) reside in the granule cell layer of the cerebellar cortex and project predominantly to the granule cells, where they form inhibitory synapses. Therefore, they reside in and project to the cerebellar cortex. Granule cells (choice C) reside in the granule cell layer of the cerebellar cortex and send their axons to the moleculary layer, where they bifurcate into parallel fibers. Granule cells, therefore, both reside in and project to the cerebellar cortex. Mossy fibers (choice D) consist of all of the afferents (except the climbing fibers) to the cerebellum. These fibers project to deep cerebellar nuclei and glomeruli in the granular layer of the cerebellar cortex.

A child who has had abnormal development of the membranous bones has a broad skull with associated facial and dental anomalies. Which other bones are most likely to also be affected? A. Clavicles B. Femurs C. Metatarsals D. Phalanges E. Tibias

The correct answer is A. In a syndrome called cleidocranial dysostosis, absence of part of the clavicles accompanies a broad skull and facial and dental anomalies. This syndrome affects bones formed by intramembranous ossification. The femurs (choice B), metatarsals (choice C), phalanges (choice D), and tibias (choice E) are cartilaginous (formed by endochondral ossification) rather than membranous bones.

If cirrhosis causes obstruction of the portal circulation within the liver, portal blood could still be conveyed to the caval system via which of the following?

A. Azygos and hemiazygos veins B. Gonadal veins C. Internal iliac veins D. Splenic vein E. Vesical venous plexus

The correct answer is A. The esophageal venous plexus, which drains into the azygos and hemiazygos veins within the thorax, has anastomoses with branches of the left gastric vein. Thus, following blockage of the portal vein, portal blood may enter the superior vena cava via the azygos system. Other important portacaval connections include the superior rectal vein with the middle and inferior rectal veins; paraumbilical veins and epigastric veins (engorgement of these vessels results in caput medusae); and the colic and splenic veins with renal veins and veins of the posterior body wall. The gonadal veins (choice B) exclusively drain the gonads (although in the female, the ovarian vein communicates with the uterovaginal plexus). These vessels have no anastomoses with portal veins. The internal iliac veins (choice C), which drain most of the pelvis and much of the inferior extremities, have no demonstrated portal anastomoses. The splenic vein (choice D) is incorrect because it's in itself a component of the portal venous system. The vesical venous plexus (choice E), which is situated well within the pelvis and drains the bladder and the prostate (or uterus and vagina) has no association with portal vessels.

Secretory basket cells are located A. inside secretory acini B. surrounding the intercalated duct C. between secretory cells and the basal lamina D. surrounding the striated duct

The correct answer is C. Secretory basket cells are found in salivary gland acini and are also called myoepithelial cells. The name implies that they are epithelial cells with muscle-like contractile properties. The are found between secretory cells and the basal lamina and exert pressure on the secretory cells to release their secretions. Secretory acini themselves are composed of mucous or serous secreting cells. Intercalated duct cells are nonspecialized cuboidal epithelial cells. Striated duct cells are columnar epithelial cells with striations (stripes) of rows of mitochondria used for ATP generation for active transport.

Which muscle is the chief mover of the mandible TOWARD the left? A. Left medial pterygoid B. Left lateral pterygoid C. Right medial pterygoid D. Right lateral pterygoid

The correct answer is D. There are two methods for answering this question. The formula method states that the right lateral pterygoid moves the mandible left, whereas the left lateral pterygoid moves the mandible right. The understanding method begins with the idea that the medial pterygoid is a closer (elevator). The lateral pterygoid connects to the condyle and is a protruder. If the right lateral pterygoid pulls the right condyle out while the left lateral pterygoid and condyle remain stable, the left side will act as a stationary pivot point while the right side protrudes outward and also toward the left (medially).

Which of the following cells is the germ cell closest to the basal lamina in the seminiferous tubule? A. Primary spermatocyte B. Secondary spermatocyte C. Spermatid D. Spermatogonia E. Spermatozoa

The correct answer is D. Maturation of germ cells (spermatogenesis) within the seminiferous tubules occurs in a concentric pattern, with the less mature spermatogonia near the basal lamina and the mature forms near the tubule center. Along this route the developing sperm are nurtured by sertoli cells. Spermatogonia are 2N cells and mature into larger primary spermatocytes (4N) (choice A). These mature into secondary spermatocytes (2N) (choice B), and finally into spermatids (1N) (choice C). Spermatids undergo spermiogenesis to become mature spermatozoa (choice E). Acrosomes form from the Golgi apparatus, and a flagellum forms from microtubules. Unneeded organelles are shed. The seminiferous tubules of a reproductive-age male should exhibit all stages of maturation, with mature flagellated sperm in their centers, no longer associated with sertoli cells.

If a person has normal musculature, but has difficulty swallowing, which nerves should be tested for function? A. Hypoglossal and phrenic B. Hypoglossal and splanchnic C. Glossopharyngeal and vagus D. Phrenic and vagus E. Splanchnic and vagus

The correct answer is E. The upper 2/3 of the esophagus contains striated muscle. It is derived from the pharyngeal arches and innervated by the vagus nerve (CN X). The lower 1/3 contains smooth muscle from splanchnic mesoderm and is innervated by the splanchnic plexus. The vagus nerve has mixed sensory and motor functions. The sensory fibers innervate the sensory nuclei and autonomic centers of the medulla oblongata. The motor fibers innervate muscles of the palate, pharynx, and respiratory and cardiovascular systems. The hypoglossal nerve (choices A and B), or CN XII, moves the tongue. The phrenic nerve (choices A and D), derived from C3, C4, and C5, innervates the muscle of the diaphragm. The glossopharyngeal nerve (choice C), or CN IX, functions in taste, swallowing, and salivation,

as well as monitoring the activity of the carotid body.

The left adrenal vein drains directly into which of the following veins? A. Hemiazygos vein B. Inferior vena cava C. Left renal vein D. Splenic vein E. Superior mesenteric vein

The correct answer is C. The left adrenal vein and the left gonadal vein (either testicular or ovarian) drain into the left renal vein. The left renal vein then drains into the inferior vena cava. In contrast, the right adrenal vein and right gonadal vein drain directly into the inferior vena cava. The hemiazygos vein (choice A) receives the venous drainage from the body wall on the left side of the thorax and abdomen. No visceral organs drain directly to the azygos or hemiazygos veins. The inferior vena cava (choice B) receives the direct venous drainage from the right adrenal vein, but not the left adrenal vein. Remember, the inferior vena cava is on the right side of the abdomen. The splenic vein (choice D) receives the venous drainage from the spleen and part of the pancreas and stomach. The splenic vein is part of the portal venous system. The superior mesenteric vein (choice E) receives venous drainage from much of the intestinal tract. It is part of the portal venous system and joins with the splenic vein to form the portal vein.

Which of the following is present in males but not in females? A. Bulbospongiosus muscle B. Bulbourethral gland

C. Corpus cavernosum D. Membranous urethra E. Perineal body

The correct answer is B. The bulbourethral glands are paired structures located within the deep perineal pouch, embedded within the sphincter urethrae. Their ducts pass to the spongy urethra. The homologous female structures are the greater vestibular (Bartholin's) glands, which are located in the superficial perineal pouch. The bulbospongiosus muscles (choice A) lie superficial to the bulb of the penis in males and to the bulbs of the vestibule in females. The corpora cavernosa (choice C) are paired structures, consisting of cavernous erectile tissue that form a large portion of the penile shaft in males and of the body of the clitoris in females. The membranous urethra (choice D) is the portion of the urethra that passes through the urogenital diaphragm in both males and females. The perineal body (choice E) is the centrally located tendinous structure that provides attachment for perineal musculature in both males and females. It separates the urogenital area from the anal area and is an important obstetric landmark.

Luminal narrowing of which of the following vessels would compromise blood flow through the renal arteries? A. Abdominal aorta B. Celiac trunk C. Common iliac artery D. Inferior mesenteric artery E. Superior mesenteric artery

The correct answer is A. The renal arteries emerge from the abdominal aorta at about the level of the L1/L2 intervertebral disk and travel at nearly right angles to it (on the right, passing

posterior to the inferior vena cava) to enter the hilum of the kidney. The celiac trunk (choice B) gives off the common hepatic, splenic, and left gastric arteries. The common iliac artery (choice C) gives off the internal and external iliac arteries. In addition, an unascended pelvic kidney may be supplied by the common iliac artery. The inferior mesenteric artery (choice D) gives off the superior rectal, sigmoid, and left colic arteries. The superior mesenteric artery (choice E) gives off the inferior pancreaticoduodenal, intestinal (ileal and jejunal), right colic, middle colic, and ileocolic arteries.

Which muscle has fibers that enter the articular disc and capsule? A. Medial pterygoid B. Buccinator C. Temporalis D. Lateral pterygoid E. Masseter

The correct answer is D. Only the lateral pterygoid enters the temperomandibular joint (TMJ). Its fibers join the articular disc and capsule. These fibers are from the superior head of the muscle. Fibers from the inferior head enter the bony head of the mandibular condyle. The lateral pterygoid protrudes the mandible, depresses (opens) it, and moves it left and right. None of the other muscles listed enter the TMJ. Medial pterygoid and masseter primarily elevate (close) the mandible, whereas temporalis elevates (anterior fibers) and retrudes (posterior fibers). Buccinator forms the substance of the cheek and is also a muscle of facial expression. It is not a muscle of mastication.

While lying supine in bed eating, a child aspirates a peanut. Which of the following bronchopulmonary segments would this foreign object most likely enter?

A. Apical segment of the left upper lobe B. Apical segment of the right upper lobe C. Medial segment of the right middle lobe D. Posterior basal segment of the left lower lobe E. Superior segment of the right lower lobe

The correct answer is E. Because the right main bronchus is wider and more vertical than the left, foreign objects are more likely to be aspirated into the right main bronchus. The superior segmental bronchus of the lower lobar bronchus is the only segmental bronchus that exits from the posterior wall of the lobar bronchi. Therefore, if a patient is supine at the time of aspiration, the object is most likely to enter the superior segmental bronchus of the lower lobe. None of the segmental bronchi of the left lung (choices A and D) are likely to receive the object because the object is less likely to enter the left main bronchus. The apical segment of the right upper lobe (choice B) is not likely to receive the foreign object because of the sharp angle that the upper lobar bronchus makes with the right main bronchus and the sharp angle that the apical segmental bronchus makes with the lobar bronchus. The medial segmental bronchus of the right middle lobe (choice C) arises from the anterior wall of the right middle lobar bronchus. Therefore, when the patient is supine, the effect of gravity will tend to prevent the object from entering this segmental bronchus.

A physician notes weakness of a patient's thumb in extension, although rotation, flexion, abduction, adduction, and opposition are normal. Which of the following nerves is most likely involved? A. Median and radial B. Median and ulnar C. Median only D. Radial only E. Ulnar only

The correct answer is D. All three of the nerves listed innervate muscles that supply the thumb. Extension is provided by the extensors pollicis longus and brevis, which are innervated by the radial nerve. The median nerve (choices A, B, and C) supplies the thenar group, which allows the thumb to oppose, flex, abduct, and rotate. The ulnar nerve (choices E and B) supplies the adductor pollicis, which adducts the thumb.

From which of the following arteries does the sphenopalatine artery arise? A. External carotid B. Facial C. Maxillary D. Ophthalmic E. Transverse facial

The correct answer is C. The sphenopalatine artery is the terminal branch of the maxillary artery. The maxillary artery arises from the external carotid artery and then passes through the infratemporal fossa, giving off branches to structures in this region. The artery then passes through the pterygomaxillary fissure to enter the pterygopalatine fossa. The terminal branch then passes through the sphenopalatine foramen to enter the nasal cavity and supply much of the nasal mucosa, particularly in the posterior region of the nasal cavity. The external carotid artery (choice A) arises from the common carotid artery at the carotid bifurcation. The external carotid artery has eight branches that supply the head and neck region: the superior thyroid, ascending pharyngeal, occipital, lingual, facial, posterior auricular, maxillary, and superficial temporal arteries. The facial artery (choice B) arises from the external carotid artery. It provides blood supply to much of the facial region. The nasal cavity (particularly the anterior portion) receives some of its blood supply from branches of the facial artery. The ophthalmic artery (choice D) arises from the internal carotid artery immediately after the internal carotid artery emerges from the cavernous sinus. The ophthalmic artery passes through

the optic canal to supply structures in the orbit. The transverse facial artery (choice E) is a branch of the superficial temporal artery. It supplies blood to structures in the upper portion of the lateral face.

Which of the following helps to anchor an epithelial cell to the basement membrane? A. Adherent junction B. Connexon C. Gap junction D. Hemidesmosome E. Tight junction

The correct answer is D. Desmosomes are specializations of the lateral surface of the cell formed from the juxtaposition of two disc-shaped structures in adjacent cells, acting much like spot welds between the two cells. Hemidesmosomes are basically desmosomes between a single cell and an extracellular matrix structure, such as a basement membrane. Adherent junctions (choice A), or zonula adherens are bandlike junctions that help attach adjacent epithelial cells to each other. Gap junctions consist of a hexagonal lattice of tubular proteins called connexons (choice B), which form channels allowing communication between cells. Tight junctions (choice E), or zonula occludens, are formed by the fusion of the outer leaflets of apposed cell membranes on the lateral cell surfaces, just beneath the apical poles. They form a barrier to permeability, or a seal around the cell.

A patient is involved in an accident which tears the left lateral pterygoid muscle completely. On attempting to open, the patients mandible will move: A. Left B. Right

C. In an elevating direction D. In a direct protruding direction

The correct answer is A.There are two ways to answer this question. The formulaic method says that damage to a lateral pterygoid or the innervation to it will make the mandible deviate toward the side of the damage. A deeper understanding of the question would show that the lateral pterygoid is the primary protruder of the mandible. Contraction of both lateral pterygoids produces straight protrusion. If the left lateral pterygoid is torn, the right muscle will begin to protrude the right side, with the left side remaining stable, and acting as a stationary pivot point. As a result, the mandible will pivot out and toward the left (the side of injury). Note that damage to muscles or innervation to one side of a tongue will also produce deviation toward the injured side.

Which of the tongue papillae are NOT vascular? A. Fungiform B. Filiform C. Vallate D. Foliate

The correct answer is B. The four major lingual papillae types are listed in the answer. Fungiform papillae are fairly numerous, vascular, have taste buds, and are found primarily on the anterior tongue. Filiform are avasacular, without taste buds, are the most numerous, and are found in rows. Vallate are largest, usually 7-9 in number, contain taste buds, and serous salivary glands of Von Ebner. Foliate are on the lateral surface of the tongue in ridges but are usually considered rudimentary and nonfunctional in man.

Mucopurulent exudate from maxillary sinusitis would be most likely to drain through an ostium in the

A. bulla ethmoidalis B. hiatus semilunaris C. inferior nasal meatus D. sphenoethmoidal recess E. superior nasal meatus

The correct answer is B. This patient has two risk factors for sinusitis: chronic rhinitis and allergy. She probably also has aspirin allergy, which is associated with the triad of nasal polyps, asthma, and sinusitis. In maxillary sinusitis, exudate may drain into the middle meatus through an ostium in the hiatus semilunaris, which contains openings to the frontal and maxillary sinuses and anterior ethmoidal cells. The bulla ethmoidalis (choice A), also part of the middle meatus, contains an opening to the middle ethmoidal air cells. The inferior nasal meatus (choice C) receives fluid from the nasolacrimal duct, which drains tears from the medial aspect of the orbit to the nasal cavity. The sphenoethmoidal recess (choice D) is located above the superior concha and contains an opening for the sphenoid sinus. The superior nasal meatus (choice E) is located above the superior concha and contains an opening for the sphenoid sinus.

A wound to the posterior left axillary line, between the ninth and tenth rib, and extending approximately 5 cm deep, will most likely damage which organ? A. Ascending colon B. Duodenum C. Left kidney D. Left lobe of the liver

E. Spleen

The correct answer is E. The spleen follows the long axes of ribs 9 to 11 and lies mostly posterior to the stomach, above the colon, and partly anterior to the kidney. It is attached to the stomach by a broad mesenterial band, the gastrosplemic ligament. Therefore, it is the most likely organ of the group to be pierced by a sharp object penetrating just above rib 10 at the posterior axillary line. Note that the pleural cavity, and possibly the lower part of the inferior lobe of the lung, would be pierced before the spleen. The ascending colon (choice A) is on the wrong side (the right) to be penetrated by a sharp instrument piercing the left side. Most of the duodenum (choice B) is positioned too far to the right to be affected by this injury. Even the third part of the duodenum, which runs from right to left, would still be out of harm's way. In addition, the duodenum lies at about levels L1 to L3, placing it too low to be injured in this case. The superior pole of the left kidney (choice C) is bordered by the lower part of the spleen. However, it is crossed by rib 12 and usually does not extend above rib 11. It would probably be too low and medial to be injured in this case because this penetration is at the posterior axillary line. The left lobe of the liver (choice D) is positioned just beneath the diaphragm, just over and anterior to the stomach. The anterior positioning of this structure makes it an unlikely candidate for injury in this case. Even with deep penetration at the correct angle, it would not be penetrated before the spleen.

You might also like